You are on page 1of 55

Antepartal Care

A nursing instructor asks a nursing student to describe the procedure for performing
the Heimlich maneuver on an unconscious pregnant woman at 8 months' gestation.
The student describes the procedure correctly if the student states that she or he will:

1. Place the hands in the pelvis to perform the thrusts.


2. Perform abdominal thrusts until the object is dislodged.
3. Perform left lateral abdominal thrusts until the object is dislodged.
4. Place a rolled blanket under the right abdominal flank and hip area.

To perform the Heimlich maneuver on an unconscious woman in an advanced stage


of pregnancy, place the woman on her back. Place a wedge, such as a pillow or rolled
blanket, under the right abdominal tlank and hip Ic displace the uterus to the left side
of the abdomen-Options 1, 2. and 3 are incorrect and can harm the woman and the
fetus.

A nurse is providing information to a client on the female reproductive system. While


discussing the uterus and its different layers, the nurse understands that the
myometrium has unique muscle fibers that make it ideally suited for the birth process.
The nurse would describe this layer to the client as:

1. The inner layer of muscle lhal is in the uterus


2. The middle layer of thick muscle in the uterus
3. The outer layer of muscle that covers most of the uterus
4. The functional layer that lies above the basal layer of the uterus

The myometrium is the middle layer of thick muscle in the uterus. These muscles
assist the birth process by expelling the fetus, ligating blood vessels after birth, and
controlling the opening of the cervical os. Options 1,3, and 4 describe the other layers
of the uterus.

A pregnant client tells the clinic nurse that she wants lo know the sex of the baby as
soon as it can be determined. The nurse knows that the client should be able to find
out at 12 weeks' gestation because by the end of the twelfth week:

1. The sex of the baby can be determined by Ihe appearance of the external
genitalia.
2. The sex of the baby can be determined because the external genitalia begin to
differentiate.
3. The sex of the baby can be determined because the testes are descended into the
scrotal sac.
4. The sex of the baby can be determined because the internal differences in males
and females become apparent.
By the end of the Iwelflh week, Ihe external genitalia o( the fetus have developed to
such a degree that the sex of Ihe fetus can be determined visually. Option 2 occurs at
the end of the ninth week. Option 3 occurs at the end of the thirty-eighth week. Option
4 occurs at the end of the seventh week.

A nurse is collecting data during an admission assessment of a client who is pregnant


with twins. The client has a healthy 5-year-old child that was delivered at 38 weeks
and tells the nurse that she does not have a history of any type of abortion or fetal
demise. The nurse would document the GTPAL for this client as

1.G-3.T-2, P-O.A-O.L-1
2.G-2.T=1.P-0.A-O.L-1
3.G=1.T=1.P=1,A = 0.L=1
4. G = 2, T = 0, P = 0, A = 0. L-1

Pregnancy outcomes can be described wilh Ihe acronym GTPAL G is gravidity, the
number of pregnancies. 7" is term births. Ihe number bom at term (longer than 37
weeks), P is preterm births, the number bom before 37 weeks' gestation, A is
abortions or miscarriages, the number of abortions or miscarriages (included in
gravida if before 20 weeks' gestation; included in parity if past 20 weeks' gestation),
and L is the number of current living children. Therefore, a woman who is pregnant
with twins and has a child has a gravida of 2. Because the child was delivered at 38
weeks, the number of term births is 1 and the number ofpreterm births is 0. The
number of abortions is 0 and the number of living children is 1.

A pregnant client is seen in a health care clinic for a regular prenatal visit. The client
tells the nurse that she is experiencing irregular contractions, and the nurse determines
that she is experiencing Braxton Hicks contractions. Based on this finding, which
nursing action is appropriate?

1. ConlacI Ihe physician.


2. Instruct Ihe client to maintain bed rest for the remainder of the pregnancy.
3. Inform the client that these are common and may occur throughout the
pregnancy.
4. Call the maternity unit and inform them that the client will be admitted in a
pretabor condition.

Braxlon Hicks conlraclions are irregular, painless contractions that may occur
intermittently throughout pregnancy. Because Braxton Hicks contractions may occur
and are normal in some pregnant women during pregnancy, options 1, 2, and 4 are
unnecessary and inappropriate actions.

A nurse is reviewing the record of a client who has just been told that a pregnancy test
is positive. The physician has documented the presence of Goodell's sign. The nurse
determines that this sign indicates:

1. A softening of the cervix


2. The presence of felal movement
3. The presence of human chorionic gonadotropin in the urine
4. A soft blowing sound that corresponds to the maternal pulse during auscultation of
the uterus

In the early weeks o( pregnancy. Ihe cervix becomes softer as a result of increased
vascularity and hyperplasia, which cause Goodell's sign. Cervical softening is noted
by the examiner during pelvic examination. A soft blowing sound that corresponds to
the maternal pulse may be auscultated over the uterus and is caused by blood
circulating through the placenta. Human chorionic gonadotropin is noted in maternal
unne in a positive urine pregnancy test. Goodell's sign does not indicate the presence
of fetal movement.

A pregnant client visits a clinic for a scheduled prenatal appointment. The client tells
the nurse that she frequently has a backache, and the nurse provides instructions
regarding measures that will assist in relieving the backache. Which statement by the
client indicates a need for further instructions?

1. "I should wear flal-heeled shoes."


2. "I should sleep on a firm matlress.'
3. "I should try to maintain good posture."
4. "I should do more exercises to strengthen my back muscles."

Some measures (hat will assist in relieving a backache include maintaining good
posture and body mechanics, resting and avoiding fatigue, wearing flat-heeled shoes,
and sleeping on a firm mattress. The back discomfort that occurs in a pregnant client
is often caused by the exaggerated lumbar and cervicothoracic curves resulting from a
change in the center of gravity because of the enlarged uterus. Performing more
exercises to strengthen the back muscles could be harmful to a pregnant client.

A nurse is providing instructions to a pregnant client who is scheduled for an


amniocentesis. The nurse tells the client that:

1. Strict bed rest is required following the procedure.


2. An informed consent will need to be signed before the procedure.
3. Hospitalization is necessary for 24 hours following the procedure.
4. A fever is expected following the procedure because of the trauma to the abdomen.

amniocentesis is an invasive procedure, informed consent will need to be obtained


before the procedure. After the procedure, Ihe client is instructed lo rest but may
resume light activity after the camping subsides. The client is instructed to keep the
puncture site clean and to report any complications such as vaginal discharge, severe,
persistent cramping, or onset of fever. Amniocentesis is an outpatient procedure and
may be done in a physician's pnvale office or in a special prenatal testing unit.
Hospitalization is not necessary following the procedure.

A pregnant client in the first trimester calls a nurse at a health care clinic and reports
that she has noticed a thin, colorless, vaginal drainage. The nurse should make which
statement to the client?

1. "Come to Ihe clinic immediately."


2. "Report to the emergency room at the maternity center immediately.
3. The vaginal discharge may be bothersome but is a normal occurrence."
4. "Use tampons if the discharge is bothersome but to be sure to change the tampons
every 2 hours.*

Leukorrhea begins during the first trimester. Many clients notice a thin colorless or
yellow vaginal discharge throughout pregnancy. Some clients become distressed
about this condition, but it does not require that the client report to the health care
clinic or the emergency room immediately. If vaginal discharge is profuse, the client
may use panty liners but should not wear tampons because of the risk of infection. If
the client uses panty liners, she should change them frequently.

A physician has prescribed transvaginal ultrasonography for a client in the first


trimester of pregnancy and the client asks the nurse about the procedure. The nurse
tells the client that:

1. The procedure lakes aboul 2 hours.


2. II will be necessary to drink 1 to 2 quarts of water before the examination.
3. Gel is spread over the abdomen, and a round disk transducer will be moved over
the abdomen to obtain the picture.
4. The probe that will be inserted into the vagina will be covered with a
disposable cover and coated with a gel.

Transvaginal ullrasonography allows clear visibility of the uterus, gestational sac,


embryo, and deep pelvic structures, such as the ovaries and fallopian tubes. The client
is placed in a lithotomy position and a transvaginal probe, encased in a disposable
cover and coated with a gel that provides lubrication and promotes conductivity, is
inserted into the vagina. The client may feel more comfortable if she is allowed to
insert the probe. The procedure takes about 10 to 15 minutes. Options 2 and 3 identify
components of the abdominal ultrasound.

A pregnant client calls a clinic and tells the nurse that she is experiencing leg cramps
that awaken her at night. To provide relief from the leg cramps, the nurse tells the
client to:

1. Bend her fool toward her body while flexing the knee when Ihe cramps occur.
2. 8end her fool toward her body while extending the knee when the cramps
occur.
3. Point her foot away from her body while flexing the knee when the cramps occur.
4. Point her foot away from her body while extending the knee when the cramps
occur.

Leg cramps occur when Ihe pregnant client stretches her leg and plantar flexes her
foot. Dorsiflexion of the foot while extending the knee stretches the affected muscle,
prevents the muscle from contracting, and stops the cramping. Options 1, 3, and 4 are
not measures that will provide relief from the leg cramps.

A clinic nurse is providing instructions to a pregnant client regarding measures that


will assist in alleviating heartburn. Which statement by the client indicates an
understanding of the instructions?
1. "I should avoid between-meal snacks."
2. "I should lie down for an hour after eating."
3. "I should use spices for cooking rather than using salt."
4. "I should avoid eating foods that produce gas, such as beans, vegetables, and
fatty foods like deep fried chicken."

Lying down is likely to lead to reflux of stomach contents, especially immediately


following a meal. The client should be instructed to avoid spices, along with salt,
because spices will trigger heartburn. Salt will produce edema. The client should be
encouraged to eat belween-meal snacks and should be instructed that to control
heartburn, eating smaller, more frequent portions is preferred over eating three large
meals. The client also should limit or avoid gas-producing and fatty foods.

A nurse in a health care clinic is instructing a pregnant client about how to perform
"kick counts." Which statement by the client indicates a need for further instructions?

I 1. "I will record Ihe number of movemenls or kicks."


I 2. "I need to lie flat on my back to perform the procedure."
\ 3. 'A count of fewer than 10 kicks in a 12-hour period indicates the need to contact
the physician."
\ 4. "I should place my hands on the largest part of my abdomen and concentrate on
the fetal movements to count the kicks."

The clienl should sit or lie quietly on her side to perform kick counts. Lying flat on
the back is not necessary to perforin this procedure, can cause discomfort, and
presents a risk of vena cava (hypotensive) syndrome. The client is instructed to place
her hands on the largest part of the abdomen and concentrate on the fetal movements.
The client records the number of movements felt during a specified time period. The
client needs to notify the physician or nurse-midwife if there are fewer than 10 kicks
in a 12-hour period or as instructed by the physician or nurse-midwife.

A pregnant client reports to a health care clinic, complaining of loss of appetite,


weight loss, and fatigue. Following assessment of the client, tuberculosis is suspected.
A sputum culture is obtained and identifies Mycobacterium tuberculosis. The nurse
provides instructions to the client regarding therapeutic management of the
tuberculosis and the nurse tells the client that:

i 1. Therapeutic abortion Is required.


i 2. She will have to stay at home until treatment is completed.
i 3. Medication will not be started until after delivery of the fetus.
i 4. Isoniazid (INH) plus rifampin (Rifadin) will be required for a total of 9
months.

More lhan one medication may be used to prevent the growth of resistant organisms
in the pregnant client with tuberculosis. Treatment must continue for a prolonged
penod of time. The preferred treatment for the pregnant client is isoniazid (INH) plus
rifampin (Rifadin) daily for a total of 9 months. Ethambutol is added initially if
medication resistance is suspected. Pyridoxine (Vitamin Be) often is administered
with isoniazid to prevent fetal neurotoxicity. It is not necessary for the client to slay at
home during treatment and therapeutic abortion is not required.

During a prenatal visit, the nurse is explaining dietary management to a client with
diabetes mellitus. The nurse determines that teaching has been effective if the client
makes which statement?

1. "Diet and Insulin needs change during pregnancy."


2. "I will plan my dlel based on the results of urine glucose testing."
3. "I will need to eat 600 more calories every day since I am pregnant.
4. "I can continue with the same diet as before pregnancy, as long as it is well-
balanced."

Tie diet for a pregnanl client with diabetes mellitus is individualized to allow for
increased fetal and metabolic requirements, with consideration of such factors as
prepregnancy weight and dietary habits, overall health, ethnic background, lifestyle,
stage of pregnancy, knowledge of nutrition, and insulin therapy. An increase of 600
additional calories a day is not required. Diet and insulin needs change during the
pregnancy in direct correlation to hormonal changes and energy needs. In the third
trimester, insulin needs increase. Dietary management during diabetic pregnancy must
be based on blood, not unne. glucose changes.

A nurse is providing instructions to a maternity client with a history of cardiac disease


regarding appropriate dietary measures. Which statement, if made by the client,
indicates an understanding of the information provided by the nurse?

1. "I should drink adequate fluids and increase my intake of high-fiber foods."
2. "I should maintain a low-calorie diet to prevent any weight gain."
3. "I should lower my blood volume by limiting my fluids."
4. "I should increase my sodium intake during pregnancy."

Constipation can cause Ihe client to use Valsalva's maneuver. This maneuver can
cause blood to rush to the heart and overload the cardiac system. Therefore, high-fiber
foods are important. A low-calorie diet is not recommended during pregnancy and
could be harmful to the fetus. Oiets low in fluid can cause a decrease in blood volume,
which could deprive the fetus of nutrients so adequate fluid intake and high-fiber
foods are important. Sodium should be restricted somewhat, as prescribed by the
physician, because excess sodium will cause an overload to the circulating blood
volume and contribute to cardiac complications.

A clinic nurse is performing a psychosocial assessment of a client who has been told
that she is pregnant. Which assessment finding indicates to the nurse that the client is
at high risk for contracting human immunodeficiency virus (HIV)?

1. A client who has a history of intravenous drug use


2. A client who has a significant other who is heterosexual
3. A client who has a history of sexually transmitted diseases
4. A client who has had one sexual partner for the past 10 years
Human immunodeficiency virus (HIV) is transmitted by intimate sexual contact and
the exchange of body fluids, exposure to infected blood, and passing from an infected
woman to her fetus. Clients who fall into the high-risk category for HIV infection
include those with persistent and recurrent sexually transmitted diseases, a history of
multiple sexual partners, or have used intravenous drugs. A heterosexual partner,
particularly a partner who has had only one sexual partner in 10 years, does not have a
high risk for contracting HIV.

A nurse assists a pregnant client with cardiac disease to identify resources to help her
care for her 18-month-old child during the last trimester of pregnancy. The nurse
encourages the pregnant client to use these resources primarily to:

1. Reduce excessive maternal stress and fatigue.


2. Help the mother prepare for labor and delivery.
3. Avoid exposure to potential pathogens and resulting infections.
4. Prepare the 18-month-old child for maternal separation during hospitalization.

A variety of (actors can cause increased emotional stress during pregnancy, resulting
in further cardiac complications. The client with known cardiac disease is at greater
risk for such complications. The use of resources will assist the client to avoid
emotional stress, thus reducing additional cardiac compromise during the last
trimester. These resources are not intended to minimize potential risk of maternal
infection or prepare the client and family for the subsequent labor, delivery, and
hospitalization.

A nurse is performing an initial assessment on a client who has just been told that a
pregnancy test is positive. Which assessment finding would indicate that the client is
at risk for preterm labor?

1. The client is a 35-year-old pnmigravida.


2. The clienl has a history of cardiac disease.
3. The client's hemoglobin level is 13.5 g/dL.
4. The client is a 20-year-old primigravida of average weight and height.

Several factors are associated with preterm labor. These include a history of medical
conditions, present and past obstetric problems, social and environmental factors, and
demographic factors such as race and age. Other risk factors include a multifetal
pregnancy, which contributes to overdistenlion of the uterus, anemia, which decreases
oxygen supply to the uterus, and age younger 18 years or first pregnancy older than
the age of 40.

The nurse is interviewing a 16-year-old client during her initial prenatal clinic visit.
The client is beginning week 18 of her first pregnancy. Which statement, if made by
the client, indicates an immediate need for further investigation?

1. "I don't like my face any more. I always look like I have been crying."
2. "I don't like my breasts anymore. These silver lines are ugly."
3. "I don't like my stomach anymore. That brown line is disgusting."
4. "I don't like my figure anymore. My clothes are all too tight."
In oplion 1, (here is an implication of periorbital and facial edema, which could be
indicative of gestational hypertension. Because the question identifies an adolescent
who has not sought early prenatal care, she is at higher risk for the development of
gestational hypertension. Options 2, 3. and 4 also deal with body image and, although
these comments should not be ignored, the need for follow-up is not urgent.

The client seen in the health care clinic has tested positive for gonorrhea. The nurse
anticipates that which medication will be prescribed for the client based on this
finding?

1. Ceftriaxone (Rocephin)
2. Penicillin G benzathine (Btcillin L-A)
3. Acyclovir (Zovirax)
4. Azithromycin (Zithromax)

Trealment for gonorrhea consists of antibiotic therapy with ceftriaxone, plus oral
doxycycline, for 7 days; therefore, option 1 is correct. Option 2 is the trealment for
syphilis, option 3 is the treatment for genital herpes simplex virus, and option 4 is the
treatment for chlamydia.

The nurse reviews the plan of care for a client at 37 weeks of gestation who has sickle
cell anemia. The nurse determines that which nursing diagnosis listed on the nursing
care plan will receive the highest priority?

1. Coping, ineffective
2. Body image, disturbed
3. Risk for pain, acute
4. Fluid volume, deficient

For the client with sickle cell anemia, dehydration will precipitate sickling of the red
Wood cells. Sickling can lead to life-threatening consequences for the pregnant
woman and for the fetus, such as an interruption of blood flow to the placenta.
Options 1. 2, and 3 may also be appropriate nursing diagnoses for the client with
sickle cell anemia but are not the priority.

The nurse provides instructions to a malnourished client regarding iron


supplementation during pregnancy. Which statement, if made by the client, would
indicate an understanding of the instructions?

i 1. The iron is best absorbed if taken on an empty stomach."


i 2. "Meat does not provide iron and should be avoided."
i 3. "Iron supplements will give me diarrhea."
i 4. "My body has all the iron it needs and I don't need to take supplements.*

Iron is needed both lo allow lor Iransler o( adequate iron to the fetus and to permit
expansion of the maternal R8C mass. During pregnancy, the relative excess of plasma
causes a decrease in the hemoglobin concentration and hematocrit, known as
physiological anemia of pregnancy. This is a normal adaptation during pregnancy.
Meats are an excellent source of iron. Iron supplements usually cause constipation.
Iron is best absorbed if taken on an empty stomach.

A pregnant woman in her second trimester calls the prenatal clinic nurse to report a
recent exposure to a child with rubella. Which of the following responses by the nurse
would be appropriate and supportive to the woman?

1. There is no need to be concerned if you don'I have a fever or rash within the next 2
days."
2. "Be sure to tell the doctor on your next prenatal visit, but there is little risk in the
second trimester.*
3. "You should avoid all school-age children during pregnancy.'
4. "You were wise to call. I will check your rubella titer screening results and we
can immediately identify if future interventions are needed."

Rubella virus is spread by aerosol droplet transmission through the upper respiratory
tract and has an incubation period of 14 to 21 days. The risks of maternal and
subsequent fetal infection dunng the second trimester include hearing loss and
congenital anomalies. Rubella titer determination is a standard antenatal test for
childbearing women during their initial screening and entry into the health care
delivery system. Option 4 helps clanfy maternal concerns with accurate information
based on the acquisition of rubella infection and potential fetal side effects.

A pregnant woman has a positive history of genital herpes but has not had lesions
during this pregnancy. The nurse should plan to provide which of the following
information to the client?

1. "You will be isolated from your newborn infant following delivery."


2. "You will be evaluated at the time of delivery for herpetic genital tract lesions
and, if present, a cesarean delivery will be needed."
3. There is little risk to your newborn infant during this pregnancy, birth, and
following delivery."
4. "Vaginal deliveries can reduce neonatal infection risks, even if you have an active
lesion at birth.*

Wilh active herpetic genital lesions, cesarean delivery can reduce neonatal infection
risks. In the absence of active genital lesions, vaginal delivery is indicated unless
there are other indications for cesarean delivery. Maternal isolation is not necessary,
but potentially exposed newborn infants should be cultured on the day of delivery.

A pregnant client is seen in the health care clinic and asks the nurse what causes the
breasts to change in size and appearance during pregnancy. The nurse plans to base
the response on which of the following?

1. The breast changes are because of the secretion of estrogen and progesterone.
2. The breasts become stretched because of the weight gain.
3. The increased metabolic rate causes the breasts to become larger.
4. Cortisol secreted by the adrenal glands play a factor in increasing the size and
appearance of the breasts.
During pregnancy, the breasts change in size and appearance. The increase in size is
because of the effects of estrogen and progesterone. Estrogen stimulates the growth of
mammary ductal tissue and progesterone promotes the growth of lobes, lobules, and
alveoli. A delicate network of veins is often visible just beneath the surface of the
skin. Options 2,3, and 4 are incorrect.

The nurse is conducting a prepared childbirth class and is instructing pregnant women
about the method of effleurage. The nurse instructs the women to perform the
procedure by:

1. Contracting and then consciously relaxing different muscle groups.


2. Contracting an area of the body, such as an arm or leg. and then concentrating on
letting tension go from the rest of the body.
3. Massaging the abdomen during contractions, using both hands in a circular
motion.
4. Instructing the significant other to stroke or massage a tightened muscle by the use
of touch.

Effleurage is massage of the abdomen during contractions. Women learn to do


effleurage using both hands in a circular motion. Progressive relaxation involves
contracting and then consciously releasing different muscle groups. Neuromuscular
disassociation helps the woman relax her body, even when one group of muscles is
strongly contracted. In this procedure, the woman contracts an area such as an arm or
leg and then concentrates on letting tension go from the rest of the body. Touch
relaxation helps the woman learn to loosen taut muscles when she is touched by her
partner.

During a routine prenatal visit, the client complains of gums that bleed easily with
brushing. The nurse performs an assessment and then teaches the client about proper
nutrition to minimize this problem. Which statement, if made by the client, indicates
an understanding of the proper nutrition to minimize this problem?

1. "I will eat three servings of cracked wheat bread each day."
2. "I will eat fresh fruits and vegetables for snacks and for dessert each day."
3. 'I will drink 8 oz of water with each meal."
4. "I will eat two saltine crackers before I get up each morning."

Fresh fruits and vegetables will provide vitamins and minerals needed for healthy
gums. Cracked wheat bread may abrade the tender gums: drinking water wit! meals
has no direct effect on gums, and eating saltine crackers before arising helps decrease
nausea.

A prenatal client with a history of heart disease has been instructed on care at home.
Which statement, if made by the client, would indicate that the client understands her
needs?

1. There is no reslriclion on people who visit me."


2. "I should avoid stressful situations."
3. "My weigh! gain is not important."
4. "I should rest on my right side."
Slress causes increased heart workload and Ihe client should be instructed to avoid
stress. To avoid infections, individuals with active infections should not be allowed to
visit the client. Otherwise, restrictions are not required. Too much weight gain can
place further demands on the heart. Resting should be on the left side to promote
blood return.
The nurse is reviewing the record of a pregnant client and notes that the physician has
documented the presence of Chadwick's sign. The nurse understands that the hormone
responsible for the development of this sign is which of the following?

1. Human chorionic gonadotropin (hCG)


2. Estrogen
3. Progesterone
4. Prolactin

The cervix undergoes significant changes following conception. The most obvious
changes occur in color and consistency. In response to the increasing levels of
estrogen, the cervix becomes congested with blood, resulting in the characteristic
bluish color that extends to include the vagina and labia. This discoloration, referred
to as Chadwick's sign, is one of the earliest signs of pregnancy.

A contraction stress test is scheduled for the pregnant client and the client asks the
nurse about the test. The nurse tells the client that:

1. Small amounts of oxytocin (Pltocln) are administered dunng internal fetal


monitoring to stimulate utenne contractions.
2. An external fetal monitor is attached and the woman ambulates on a treadmill until
contractions begin.
3. The uterus is stimulated to contract by small amounts of oxytocin (Pitocin) or
nipple stimulation.
4. Utenne contractions are stimulated by Leopold's maneuvers.

A contraction slress test assesses placental oxygenation and function, determines fetal
ability to tolerate labor, determines fetal well-being, and is performed if the nonstress
test is abnormal. The fetus is exposed to the stressor of contractions to assess the
adequacy of placental perfusion under simulated labor conditions. An external fetal
monitor is applied to the mother and a 20- to 30-minute baseline strip is recorded. The
uterus is stimulated to contract by the administration of a dilute dose of oxytocin
(Pitocin) or by having the mother use nipple stimulation until three palpable
contractions of 40 seconds or longer in a 10-minute period have occurred. Frequent
maternal blood pressure readings are done and the client is monitored closely while
increasing doses of oxytocin are given. Options 1, 2, and 4 are inaccurate.

A nonstress test is performed on a client who is pregnant and the results of the test
indicate nonreactive findings. The physician prescribes a contraction stress test. The
test is performed and the nurse notes that the physician has documented the results as
negative. The nurse interprets this finding as indicating:

1. A high risk for fetal demise.


2. A normal lest result.
3. The need for a cesarean delivery.
4. An abnormal lest result.

Contraction stress test results may be interpreted as negative (normal), positive


(abnormal), or equivocal. A negative test result indicates that no late decelerations
occurred in the fetal heart rate, although the fetus was stressed by three contractions
of at least 40 seconds duration in a 10-minute period. Repetitive late decelerations
render the test results positive.

A fasting blood glucose screening is performed on a pregnant client. The results


indicate that the blood glucose level is 140 mg/dL. Which of the following would the
nurse anticipate to be prescribed for the mother?

1. An oral hypoglycemic agenl


2. NPH insulin on a daily basis
3. A 3-hour glucose tolerance lest
4. A sliding scale Regular insulin dose

A maternal glucose level is determined to screen for gestational diabetes. A 50-g oral
glucose load may be prescribed and is followed by a serum glucose determination 1
hour later. If the test is given without regard for fasting, 140 mg/dL is the upper limit
of normal. If the test is given when the woman is fasting, the upper acceptable limit is
135 mg/dL. Clients exceeding these limits should be further evaluated with a 3-hour
glucose tolerance lest (GTT). Options 1, 2, and 4 would not be prescribed based
solely on the maternal glucose levels. Further follow-up would be implemented.

The pregnant client seen in the health care clinic has tested positive for human
immunodeficiency virus (HIV). Based on this information, the nurse determines that:

1. The client has the herpes simplex virus


2. HIV antibodies are detected by the ELISA test.
3. The neonate will definitely develop this disease after birth.
4. This client has contacted an airborne disease.

Diagnosis of HIV infection depends on serological studies to detect HIV antibodies.


The most commonly used test is the enzyme-linked immunosorbent assay (ELISA).
Options 1 and 4 are incorrect because HIV infection primarily occurs through the
exchange of body fluids. Option 3 is incorrect. A neonate bom to an HIV-positive
mother is at risk for developing the virus.

The nursing instructor asks the nursing student about the physiology related to the
cessation of ovulation that occurs during pregnancy. Which of the following
responses, if made by the student, indicates an understanding of this physiological
process?

1. "Ovulation ceases during pregnancy because the circulating levels of estrogen


and progesterone are high."
2. "Ovulation ceases during pregnancy because the circulating levels of estrogen and
progesterone are low."
3. The low levels of estrogen and progesterone increase the release of the follicle-
stimulating hormone and luteinizing hormone.*
4. The high levels of estrogen and progesterone promote the release of the follicle-
stimulating hormone and luteinizing hormone.*

Ovulation ceases during pregnancy because Ihe circulating levels of estrogen and
progesterone are high, Inhibiting the release of follicle-stimulating and luteinizing
hormones, which are necessary for ovulation. Options 2, 3, and 4 are incorrect.

The nurse encourages the pregnant human immunodeficiency virus (HlV)-positive


client to report any early signs of vaginal discharge or perineal tenderness to the
health care providers immediately. The client asks the nurse about the importance of
this action and the nurse responds by telling the client that this is necessary to:

1. Relieve anxiety for Ihe pregnant client.


2. Eliminate the need for further unnecessary screenings.
3. Assist in identifying potential infections that may need to be treated.
4. Minimize the financial cost of caring for an HIV-positive client.

The HIV-compromised clienl may be al high risk for superimposed infections during
pregnancy. These include Candida infections, genital herpes, and anogenital
condyloma. Early reporting of symptoms may alert the members of the health care
team that further assessment and testing are needed to diagnose and manage
additional maternal and fetal physiological risks. Options 1, 2, and 4 do represent
possible outcomes of this nursing intervention, but are not the priority of care when
promoting maternal-fetal well-being.

The nurse is performing an assessment on a pregnant client at 16 weeks of gestation.


On assessment, the nurse would expect that the fundus of the uterus would be located
at which of the following areas?

1. Midway between the symphysis pubis and Ihe umbilicus


2. AI Ihe umbilicus
3. Just above the symphysis pubis
4. At the level of the xiphoid process

At 12 weeks' geslation. the uterus extends out of the maternal pelvis and can be
palpated above the symphysis pubis. At 16 weeks' gestation, the fundus reaches
midway between the symphysis pubis and umbilicus. At 20 weeks' gestation, the
fundus is located at the umbilicus. By 36 weeks' gestation, the fundus reaches its
highest level at the xiphoid process.

In the prenatal clinic, the nurse is interviewing a new client and obtaining health
history information. The nurse plans to do which of the following to elicit most
appropriately accurate responses to the questions that refer to sexually transmitted
diseases?

1. Establish a therapeutic relationship.


2. Use specific close-ended questions.
3. Omit these types of questions because they are highly personal.
4. Apologize for the embarrassment that these questions will cause the client.

The initial assessment interview establishes the therapeutic relationship between the
nurse and the pregnant woman. It is planned purposeful communication that focuses
on specific content. Options 2, 3, and 4 are incorrect and would not lend themselves to
eliciting accurate information from the client.

The maternity nurse is describing the ovarian cycle to a group of nursing students.
The instructor asks a nursing student to identify the phases of the cycle. Which of the
following, if identified as a phase of the cycle by the nursing student, indicates a need
to further research this area?

1. Follicular phase
2. Ovulatory phase
3. Luteal phase
4. Proliferative phase

The ovanan cycle consists of three phases, the follicular, ovulatory, and luteal phases.
The proliferative phase is a phase of the endometrial cycle.

A pregnant client in the prenatal clinic is scheduled for a biophysical profile. The
client asks the nurse what this test involves. The appropriate nursing response is
which of the following?

1. This test only measures the amount of amniotic fluid present in the uterus."
2. This test measures amniotic fluid volume and fetal activity."
3. This test measures your ability to tolerate the pregnancy."
4. This test measures your cardiac status and ability to tolerate labor."

Tie biophysical profile assesses five parameters of fetal activity: fetal heart rate
(FHR), fetal breathing movements, gross fetal movements, fetal tone, and amniotic
fluid volume. In a biophysical profile, each of the five parameters contributes 0 to 2
points, with a score of 8 being considered normal and a score of 10 perfect. Results
are available immediately. Options 1,3. and 4 are incorrect.

A nurse in the prenatal clinic is taking a nutritional history from a 16-ycar-old


pregnant adolescent. Which of the following statements if made by the adolescent
would alert the nurse to a potential psychosocial problem?

1. "I only want to gain 10 pounds because I want to have a small, petite baby."
2. "I will continue drinking my afternoon milkshake."
3. "I don't like dairy products."
4. "I'm not used to eating so much food, but I will try."

Pregnant adolescents are at higher nsk lor complications. Peer pressure is an


important influence on nutnlional status. Adolescents often are concerned about their
body image. If weight is a major focus for the adolescent, the adolescent is more
likely to restrict calories to avoid weight gain. Option 1 is the only option that
suggests a possible psychosocial problem. Options 2. 3. and 4 relate to physiological
issues.

A nurse in the prenatal clinic is conducting a session about nutrition to a group of


adolescents who are pregnant. The appropriate measure to teach these adolescents is
to:

1. Avoid meals in fast food restaurants.


2. Eliminate snacks during the day.
3. Monitor for appropriate weight gain patterns.
4. Eat only when hungry.

Telling an adolescent to avoid fast food restaurants and to eliminate snacks may cause
the adolescent to rebel. Advising an adolescent to eat only when hungry could lead to
a deficit in nutrients. The nurse should appropriately teach the adolescent about
appropriate weight patterns and how to monitor these patterns. The adolescent is more
likely to follow suggestions and adhere to the appropriate dietary patterns if the nurse
explains why the weight gain is important for the fetus as well as the mother.

A nurse in a health care clinic is preparing lo conduct a nutritional session with a


group of culturally diverse pregnant women. At the first session, the nurse will be
meeting with each client individually. The nurse prepares a list of items to be included
in the session and lists which item as the first priority?

I 1. Review Ihe MyPyramid food guide.


I 2. Weigh each client and ask the client to document the weight on a progress chart.
| 3. Discuss the costs of food items.
I 4. Identify the food preferences and methods of food preparation for each
client.

In order lo determine each client's nutritional status and needs, the first priority of the
nurse is to identify each client's food preferences. Cultural background and
knowledge about nutrition are important factors influencing food choices and
nutritional status. Although options 1. 2. and 3 may be a component of the sessions,
option 4 is the first priority.

A clinic nurse is discussing nutrition with a client who has lactose intolerance. The
nurse should instruct the client to supplement the dietary source of calcium by eating
which of the following foods?

1. Oned fruits
2. Creamed spinach
3. Fresh squeezed orange juice
4. Hard cheese

The best source of calcium is dairy products. Women with lactose intolerance need
other sources of calcium Calcium is present in dark, green, leafy vegetables, broccoli,
legumes, nuts, and dried fruits. Spinach contains calcium, but it also contains oxalates
that decrease calcium availability. Additionally, creamed spinach may not be tolerated
by a client with a lactose intolerance. Orange juice does not contain significant
amounts of calcium unless fortified with calcium. Cheese is a dairy product and is not
tolerated by the client with a lactose intolerance.

A nurse has provided instructions to a pregnant client who is preparing to take iron
supplements. The nurse determines that the client understands the instructions if the
client states that she will take the supplements with which of the following?

1. Milk
2. Tea
3. Orange juice
4. Coffee

Foods containing ascorbic acid will increase the absorption of iron. Calcium and
phosphorus in milk and tannin in tea decreases iron absorption. Coffee binds iron and
prevents it from being fully absorbed. Orange juice is the only item that contains
ascorbic acid and will increase the absorption of iron supplements.

A nurse is reviewing the record of a pregnant client seen in the health care clinic for
the first prenatal visit. Which of the following data if noted on the client's record
would alert the nurse that the client is at risk for developing gestational diabetes
during this pregnancy?

1. The clienl's last baby weighed 10 pounds al birth.


2. The clienl's previous deliveries were by cesarean birth.
3. The clienl has a family history of cardiovascular disease.
4. The client is 5 feel 3 inches in height and weighs 165 pounds.

Known risk factors lhal increase the nsk o( developing gestational diabetes include
obesity (over 198 pounds), chronic hypertension, family history of diabetes mellitus,
previous birth of a large infant (over 4000 g), and gestational diabetes in a previous
pregnancy. Options 2, 3. and 4 are not risk factors associated with the development of
gestational diabetes.

A nurse is teaching a diabetic pregnant client about nutrition and insulin needs during
pregnancy. The nurse determines that the client understands dietary and insulin needs
if the client states that the second half of pregnancy may require:

1. Increased caloric intake


2. Decreased caloric intake
3. Increased insulin
4. Decreased insulin

Glucose crosses the placenta, but insulin does not. Higf fetal demands for glucose,
combined with the insulin resistance caused by hormonal changes in the last half of
pregnancy, can result in elevation of maternal blood glucose levels. This increases the
mother's demand for insulin. This is referred to as the diabetogenic effect of
pregnancy. Caloric intake is not affected by diabetes.
A prenatal clinic nurse asks a coassigned nursing student to identify the physiological
adaptations of the cardiovascular system that occurs during pregnancy. The nurse
determines that the student understands these physiological changes if the student
states which of the following?

1. "A decrease in cardiac oulpul occurs."


2. "An increase in pulse occurs."
3. The systolic and diastolic blood pressures increase by 20 mm Hg."
4. "A decrease in blood volume occurs."

Between 14 and 20 weeks of gestation, the maternal pulse rate increases slowly, up to
10 to 15 beats/min, which lasts until term. Cardiac output and Wood volume increase.
Blood pressure decreases in the first half of pregnancy and returns to baseline in the
second half of pregnancy.

A nurse is using Nagele's rule to calculate a pregnant woman's estimated date of


delivery. The woman tells the nurse that her last period began on June 17, 2008, and
ended 6 days later. The nurse should compute the estimated date of delivery to be:

1. March 10. 2008


2. March 15. 2008
3. March 24. 2009
4. March 29. 2009

Nagele's rule is a noninvasive melhod for eslimaling Ihe dale of birth. The rule slates
the following: add 7 days to the first day of the last menstrual period, subtract 3
months, and add 1 year. This is based on the assumption that the cycle is 28 days.
June 17, 2008. plus 7 days minus 3 months is March 24, 2008. Adding 1 year brings
the delivery date to March 24, 2009.

A client who is 8 weeks' pregnant calls the health care clinic and tells the nurse that
she is experiencing nausea and vomiting every morning. The nurse suggests which of
the following measures that will best promote relief of Ihe symptoms?

1. Eating a high-carbohydrate diet


2. Eating a high-fat diet
3. Eating dry crackers before arising
4. Increasing fluids with meals

Some slralegies for decreasing morning nausea are keeping crackers, melba toast, or
dry cereal at the bedside to eat before getting up in the morning; eating smaller, more
frequent meals; decreasing fats; and consuming adequate fluid between meals but not
with meals. A high-carbohydrate diet could increase the episodes of nausea.

The home care nurse is visiting a prenatal client who has a history of heart disease.
The nurse provides instructions to the client regarding home care measures to promote
a healthy pregnancy. Home care for this client will include which of the following
measures?

1. Restrict visitors who may have an active Infection.


2. Avoid becoming concerned about placing stress on the heart.
3. Increase dally calories to ensure weight gain.
4. Sleep in a supine position at night.

The clienl should avoid exposure to infection and not allow those persons with active
infections to visit. Stress causes increased heart workload, with the potential for
adverse consequences. Too much weight gain causes an increase in body
requirements and increases stress on the heart. The client should rest on the left side to
promote blood return.

A couple is seen in the fertility clinic. After several tests, it has been determined that
the husband is not sterile and that the wife has nonpatent fallopian tubes. The nurse is
preparing the woman and her husband for an in vitro fertilization. Which statement by
the woman or her spouse would indicate a need for further information about the
procedure?

1. "Ova and sperm are collected and allowed to Incubate."


2. "A fertilized ovum is transferred into the woman's uterus.
3. The procedure is a method of medically assisted reproduction."
4. The procedure is performed using artificial insemination of sperm instilled
through the vagina."

tn vitro fertilization is a method of medically assisted reproduction for women with


nonpatent, diseased, or missing fallopian tubes or with infertility of unknown
etiology. Ova and sperm are obtained from the potential parent or donor, placed in a
nutrient medium, and allowed to incubate, and then the fertilized ovum is transferred
into the woman's uterus. The woman houses the pregnancy throughout gestation and
gives birth. Option 4 describes the procedure for artificial insemination. Options 1,2,
and 3 are correct statements regarding this procedure.

A nurse in the gynecological clinic is reviewing the record of a pregnant client after
the first prenatal visit. The nurse notes that the physician has documented that the
woman has a platypelloid pelvis. On the basis of this documentation, the nurse plans
care knowing that this type of pelvis:

1. Is a normal female pelvis


2. Has a flat shape
3. Has an oval shape
4. Is heart shaped

A platypelloid pelvis has a flat shape. A gynecoid pelvis is a normal female pelvis. An
anthropoid pelvis has an oval shape, and an android pelvis is heart shaped.

A prenatal clinic nurse is performing a nutritional assessment on a pregnant


adolescent. The nurse performs the assessment knowing that the primary reason why
pregnant teenagers are at risk for nulrilional deficiencies is that:

1. Their parents may be upset about the pregnancy.


2. They are still going through a growth stage.
3. They do not go to school.
4. They have missed classes on good nutrition.

Teens experience a major growth spurt, and the pregnancy adds to these normal
needs. Although all options may have an Impact on nutritional deficiencies, the
primary reason why the pregnant adolescent Is at risk for nutritional deficiencies is
the growth spurt.

A nurse is employed in a fertility clinic. A woman and her husband are seen by the
physician, and an in vitro fertilization has been discussed with the couple. The couple
ask the nurse to provide additional information about this procedure. The nurse's
explanation will appropriately include which of the following descriptions?

1. Ova and sperm are collected and Immediately transferred Into the woman's uterus.
2. A fertilized ovum Is transferred Into the woman's uterus.
3. Ova and sperm must be obtained from a donor other than the potential parents, and
it may take some time to locate the donor.
4. Artificial insemination of sperm through the vagina with a syringe commonly is
used.

tn vitro fertilization is a method of medically assisted reproduction for women with


nonpatent, diseased, or missing fallopian tubes, or with infertility of unknown
etiology. Ova and sperm are obtained from potential parents or donors, placed in a
nutrient medium, and allowed to incubate, and then the fertilized ovum is transferred
into the woman's uterus. The woman houses the pregnancy throughout gestation and
gives birth. Option 4 describes the procedure for artificial insemination. Options 1 and
3 are incorrect statements regarding this procedure.

A nurse is performing a measurement of fundal height in a client whose pregnancy


has reached 36 weeks of gestation. During the measurement, the client begins to feel
lightheaded. On the basis of knowledge of the physiological changes of pregnancy,
the nurse determines that this symptom is most likely to be due to:

1. Emotional Instability
2. Compression of the vena cava
3. A full bladder
4. Insufficient Iron intake

Compression of the inferior vena cava and aorta by the uterus may cause supine
hypotension syndrome late in pregnancy. Having the woman turn onto her left side or
elevating the left buttock dunng fundal height measurement will correct or prevent the
problem. Options 1, 3. and 4 are unrelated to this syndrome.

A pregnant client has been instructed on the prevention of genital tract infections.
Which statement by the client indicates an understanding of these preventive
measures?

1. "I should avoid the use of condoms."


2. "I can douche anytime I want."
3. "I can wear my light-filling jeans.'
4. "I should wear underwear with a cotton panel liner."

Condoms should be used to minimize Ihe spread of genital tract infections. Wearing
tight clothing can irritate the genital area and does not allow for air circulation.
Douching is to be avoided. Wearing items with a cotton panel liner allows for air
movement in and around the genital area.

A nurse is reviewing the results of the rubella screening (titer) with a pregnant 24-
year-old client. The test results are positive, and the mother asks if it is safe for her
toddler to receive the vaccine. The appropriate nursing response is:

1. "You are still susceptible to rubella, so your toddler should receive the vaccine."
2. "Most children do not receive the vaccine until they are 5 years o( age."
3. "It is not advised for children of pregnant women to be vaccinated during their
mother's pregnancy."
4. "Your titer supports your immunity to rubella, and it is safe for your toddler
to receive the vaccine at this time.*

All pregnanl women should be screened for pnor rubella exposure during pregnancy.
All children of pregnant women should receive Iheir immunizations according to
schedule. Additionally- no definitive evidence suggests that the rubella vaccine virus
is transmitted from person to person. A positive maternal titer further indicates that a
significant antibody titer has developed in response to a prior exposure to the
rubivirus.

A clinic nurse is explaining the changes in the integumentary system that occur during
pregnancy to a client and tells the client that which change may persist after she gives
birth?

1. Epulis
2. Chloasma
3. Telangiectasia
4. Striae gravidarum

Slnae gravidarum, or stretch marks, reflect separation within the underlying


connective tissue of the skin. After birth they usually fade, although they never
disappear completely. Options 1,2, and 3 are incorrect. An epulis is a red, raised
nodule on the gums that bleeds easily. Chloasma, or mask of pregnancy, is a blotchy,
browning hyperpigmentation of the skin over the cheeks, nose, and forehead and is
especially noticed in dark-complexioned pregnant women. Chloasma usually fades
after the birth. Telangiectasias, or vascular "spiders," are tiny, star-shaped or branch-
shaped, slightly raised and pulsating end-arterioles usually foundon the neck, thorax,
face, and arms. They occur as a result of elevated levels of circulating estrogen. The
"spiders" usually disappear after birth.

A clinic nurse is instructing a pregnant client regarding dietary measures to promote a


healthy pregnancy. The nurse tells the client about the importance of an adequate
intake of fluid daily. Which statement by the client indicates an understanding of the
daily fluid requirement?
1. "I should drink 12 glasses of fnjil juices and milk every day."
2. "I should drink 8 to 10 glasses of fluid a day. and I can drink as many diet soft
drinks as I want."
3. 'I should drink 12 glasses of fluid a day, and I can include the coffee or tea that I
drink in the count."
4. "I should drink at least 8 to 10 glasses of fluid each day. of which at least 6
glasses should be water.*

The nurse should instruct the client to have an adequate fluid intake daily to assist in
digestion and in the management of constipation. The pregnant client should consume
at least 8 to 10 (8-oz) glasses of fluid each day, of which at least 6 glasses should be
water. Because of their sodium content, diet soft drinks should be consumed in
moderation. Caffeinated beverages have a diuretic effect, which may be
counterproductive to increasing fluid intake. It is not necessary for the client to drink
12 glasses of fruit juices and milk every day.

A prenatal clinic nurse is providing instructions to a group of pregnant clients


regarding measures to prevent toxoplasmosis. Which statement if made by one of the
clients indicates a need for further instruction?

1. "I should cook meal thoroughly.'


2. "I should drink unpasteurized milk only."
3. "I should avoid contact with materials that are possibly contaminated with cat
feces.*
4. "I should avoid touching mucous membranes of the mouth or eyes while handling
raw meat."

All pregnant clients should be advised to do the following to prevent the development
of toxoplasmosis: Clients should be instructed to cook meats thoroughly, particularly
pork, beef, and lamb; avoid touching mucous membranes of the mouth or eyes while
handling raw meat; thoroughly wash all kitchen surfaces that come in contact with
uncooked meat; wash the hands thoroughly after handling raw meat; avoid uncooked
eggs and unpasteurized milk; wash fruits and vegetables before consumption; and
avoid contact with materials that possibly are contaminated with cat feces, such as cat
litter boxes, sand boxes, or garden soil.

A home care nurse is monitoring a 16-year-old primigravida who is at 36 weeks'


gestation with gestational hypertension. Her blood pressure during the past 3 weeks
has been averaging in the 130/90 mm Hg range. She has had some swelling in the
lower extremities and has had mild proteinuria. Which statement if made by the client
should alert the nurse to the possible development of preeclampsia in the client?

1. "My vision the pasl 2 days has been really fuzzy."


2. The swelling In my hands and ankles has gone down."
3. "I had heartburn yesterday after I ate some spicy foods."
4. "I had a headache yesterday, but I took some acetaminophen (Tylenol) and it went
away."
Visual disturbances such as blurred vision, double vision, or spots before the eyes
indicate arterial spasms and edema in the retina and may be a warning sign of
preeclampsia. A continuous headache indicates poor cerebral perfusion; having just
one headache that is relieved with medication is not an indicator of preeclampsia.
Resolution of swelling is not an indicator of preeclampsia. Heartburn is a common
discomfort of pregnancy, especially with intake of spicy foods.

A client who has just been told that she is pregnant wants to know when the baby's
heart will be completely developed and beating. The nurse reads in the client's chart
that the health care provider has determined the client to be at 6 weeks' gestation. The
nurse's best response to the client would be:

1. "Your baby's heart right now consists of Iwo parallel tubes, so we can't hear it
today."
2. "Your baby's heart right now is beginning to partition into four chambers and has
begun to beat, so we should be able to hear it with a Doppler,"
3. "Your baby's heart right now is beginning to partition into four chambers and has
begun to beat, so we should be able to hear it with a fetoscope.
4. "Your baby's heart right now has double heart chambers and has begun to
beat, so we should be able to see it beat using an ultrasound machine."

By gestational week 5, double heart chambers are visible by ultrasound and the heart
begins to beat. The fetal heart is only two parallel tubes at week 3. At week 5, the
heart can be visualized only by ultrasound. To be heard by Doppler, the gestation
must be 12 weeks; to be heard by fetoscope, the gestation must be at 20 weeks.

A clinic nurse is reviewing the chart of a client scheduled for her weekly prenatal
appointment. The nurse notes that the client has been diagnosed with mild
preeclampsia. Of the following interventions, which will the nurse list as having the
lowest priority in planning nursing care for this client?

1. Assess Wood pressure.


2. Discuss the need for hospitalization.
3. Assess deep tendon reflexes and edema.
4. Teach the importance of keeping track of a daily weight.

Wilh mild cases of preeclampsia, the condition is monitored with self-care and
bedrest while at home. The nurse must assess blood pressure, weight, and the
presence of edema because an increase in these areas would indicate a worsening
condition. Before the need for hospitalization is discussed, the client would need to be
assessed for the progression of the disease process.

A nursing instructor asks a nursing student who is preparing to assist with the
assessment of a pregnant client to describe the process of quickening. Which of the
following statements if made by the student indicates an understanding of this term?

1. "II Is the irregular, painless contractions thai occur throughout pregnancy."


2. "It is the soft blowing sound that can be heard when the uterus is auscultated."
3. "It is the fetal movement that is felt by the mother"
4. "It is the thinning of the lower uterine segment."
Quickening is fetal movemenl and may occur as early as the fourteenth to sixteenth
week of gestation; the expectant mother first notices subtle fetal movements that
gradually increase in intensity. A soft blowing sound that corresponds to the maternal
pulse may be auscultated over the uterus, and this is known as uterine souffle. This
sound is due to the blood circulation to the placenta and corresponds to the maternal
pulse. Braxton Hicks contractions are irregular, painless contractions that may occur
throughout pregnancy. A thinning of the lower uterine segment occurs about the sixth
week of pregnancy and is called Hegar's sign.

Immediately after an amniotomy has been performed, the nurse should first assess:

1. For bladder distention


2. For cervical dilation
3. The maternal blood pressure
4. The fetal heart rate (FHR) pattern

The FHR is assessed immediately after amniolomy lo delect any changes that may
indicate cord compression or prolapse. Bladder distention or maternal blood pressure
would not be the first things to check after an amniotomy. Once the membranes are
ruptured, minimal vaginal examinations will be done because of the risk of infection.

1. A multigravida at 37 weeks' gestation tells the nurse that she has frequent
heartburn. After providing the client with suggestions for obtaining relief from the
heartburn, the nurse determines that the client has understood the instructions when
she says:
A. "I can take a teaspoon of baking soda in water occasionally."
B. "I should eat only three large meals and drink plenty of fluids."
C. "It's all right for me to have a fried hamburger and fries."
D. "I should eat smaller, more frequent meals with fluids."

RATIONALE: D. The client who complains of heartburn should eat smaller, more
frequent meals with fluids. Baking soda in water should be avoided because of the
sodium in baking soda. Large meals and fried
foods should also be avoided.

2. A nurse is aware that periodic relaxation and contraction of the uterine muscles
causes pain and cramping after birth. Which client is more likely to experience severe
pain after giving birth?
A. A gravida 1 para 1001 client who's bottle-feeding
B. A gravida 2 para 2002 client who's breast-feeding
C. A gravida 3 para 1102 client who's bottle-feeding
D. A gravida 2 para 1001 client who's breast-feeding

RATIONALE: B. The client who's a second-time mother and breast-feeding will


experience more pain and cramping after giving birth. In first-time mothers, the
uterine tone is increased, so the fundus remains firm, thereby decreasing the incidence
of uterine cramping. In subsequent pregnancies, uterine tone decreases and cramping
increases as the uterine muscles contract. Mothers who breast-feed experience more
pain and cramping after giving birth. Breast-feeding initiates the release of the
hormone oxytocin from the pituitary gland. Oxytocin causes uterine contractions to be
strengthened and coordinated so that the uterus remains very firm.

4. A primipara client is 6 weeks pregnant. At her first prenatal visit, she says to the
nurse, "I just can't believe I'm really pregnant. I hope this baby is a good idea." What
should the nurse make of the client's statement?
A. The client is afraid of pregnancy and birth.
B. The client should have waited until she was committed to having a baby.
C. The client is experiencing normal ambivalence about being pregnant.
D. The client may have problems attaching to the baby after birth.

RATIONALE: C. Ambivalence is normal in the first trimester of pregnancy, even


when a pregnancy is planned and desired. Fear and attachment problems aren't
substantiated by the evidence supplied. Assuming that the client should have waited
to have her baby is judgmental.

A nurse is teaching a newly pregnant client about nausea associated with prenatal
vitamins. Which point about preventing nausea should the nurse include in her
teaching?
A. Switch brands.
B. Take the vitamin on a full stomach.
C. Take the vitamin with orange juice for better absorption.
D. Take the vitamin first thing in the morning.

RATIONALE: B. Prenatal vitamins commonly cause nausea and taking them on a


full stomach may curb this problem. Switching brands may not be helpful and may be
more costly. Orange juice tends to make pregnant women nauseated. The vitamins
may be taken at night, rather than in the morning, to reduce nausea.

14. A woman who's 10 weeks pregnant tells the nurse that she's worried about her
fatigue and frequent urination. The nurse should:
A. tell the client that these are normal early pregnancy signs and symptoms.
B. question her further about these signs and symptoms.
C. tell her that she'll need blood work and urinalysis.
D. tell her that she may be excessively worried.

RATIONALE: A. Fatigue and frequent urination are normal early signs and
symptoms of pregnancy that may continue through the first trimester. Questioning the
client about the signs and symptoms is helpful to complete the assessment but won't
reassure her. Prenatal blood work and urinalysis are routine for this situation but don't
address the client's concerns. Telling her that she may be excessively worried isn't
therapeutic.

15. Which of the following is a contributing factor to an unstable blood glucose level
in a neonate?
A. Prematurity
B. Physiological jaundice
C. Term delivery
D. Cesarean birth
RATIONALE: A. Stores of brown fat aren't deposited until 36 weeks, so infants born
at less than 36 weeks won't have the necessary stores to maintain a normal blood
glucose level. Physiological jaundice doesn't contribute to an unstable glucose level.
Neonates who are postdated, not term, will use up their stores of brown fat as a result
of these complications and may experience unstable blood glucose level. Neonates
delivered by cesarean birth without any other contributing factors should have
adequate stores of brown
fat to control the blood glucose level.

16. A client with hemolysis, elevated liver enzyme levels, and low platelet count
(HELLP) syndrome is admitted to the labor and delivery unit. The client's condition
rapidly deteriorates and, despite efforts by the staff, the client dies. After the client's
death, the nursing staff displays many emotions. Who should the nurse-manager
consult to help the staff cope with this unexpected death?
A. The human resource director, so she can arrange vacation time for the staff
B. The physician, so he can provide education about HELLP syndrome
C. The social worker, so she can contact the family about funeral arrangements
and pass along the information to the nursing staff
D. The chaplain, because his educational background includes strategies for
handling grief

RATIONALE: D. The chaplain should be consulted because his educational


background provides strategies for helping others handle grief. Providing the staff
with vacation isn't feasible from a staffing standpoint and doesn't help the staff cope
with their grief. The staff needs grief counseling, not education about HELLP
syndrome. Asking the social worker to contact the family about the funeral
arrangements isn't appropriate-

17. A client in her seventh month of pregnancy has been complaining of back pain
and wants to know what can be done to relieve it. Which response by the nurse is
most effective?
A. "You need to lie down more during the day to get off your feet."
B. "Avoid lifting heavy loads, and try using the pelvic tilt exercise."
C. "Have others pick things up for you so you don't have to bend over so much."
D. "Your back pain will go away after the baby is born."

RATIONALE: B. The pelvic tilt exercise, which can be done standing as well as lying
down, and avoiding lifting heavy loads can greatly relieve back discomfort- As the
pregnancy progresses into the last trimester, women typically develop a "swayback"
curvature of the spine to counterbalance the enlarging fetus. Tilting of the pelvis
aligns the spine, decreasing pressure and back discomfort. Lying down more during
the day may not be possible or convenient for some clients. Also, the supine position
may not be comfortable for some clients and may cause vena cava syndrome
(dizziness on rising and decreased circulation to the fetus). Avoiding bending over as
much may not be realistic for the client's circumstances, nor does it address back pain
as effectively as the pelvic tilt. Telling the client that the pain will go away after the
baby is born doesn't help to relieve the client's discomfort.

A nurse is caring for a pregnant adolescent. The client is taking an iron supplement.
What should this client drink to increase the absorption of iron?
A. A glass of milk
B. A cup of hot tea
C. A liquid antacid
D. A glass of orange juice

RATIONALE: D. Increasing vitamin C enhances the absorption of iron supplements.


Taking an iron supplement with milk, tea, or an antacid reduces the absorption of
iron.

. A community health center physician has confirmed that a 16-year-old adolescent is


at 16 weeks' gestation. The adolescent confides to the nurse that she's afraid to tell her
parents she's pregnant. Which statement best explains the client's feelings?
A. The adolescent lacks a stable relationship with her family.
B. The adolescent fears rejection by her family because of her unplanned
pregnancy.
C. The adolescent's parents may force an abortion.
D. The adolescent can't rely on her family for emotional support.

RATIONALE: B. The adolescent's perception of parental response causes fear of


rejection. Parents' initial reactions to the news of their daughter's pregnancy usually
include shock, anger, shame, guilt, and sorrow. However, in general, adaptation
occurs as the pregnancy progresses, and the adolescent's mother usually becomes her
key support system. No data in this case study indicate abnormally unstable or
nonsupportive family relationships. The evidence given is insufficient to conclude
that the client can't rely on her family for support.

A client with type 1 diabetes mellitus is pregnant for the second time. Her previous
pregnancy ended in spontaneous abortion at 18 weeks' gestation. She's now at 22
weeks' gestation. The nurse is responsible for teaching the client about exercise during
her pregnancy. Which statement indicates that the client has an appropriate
understanding of her exercise needs?
A. "I know I need to walk with a friend or family member."
B. "I know I need to vary the times of day when I exercise."
C. "I know I need to exercise before meals."
D. "I know I need to drink fluids while I walk."

RATIONALE: A. The client's statement demonstrates understanding of her exercise


needs because a client with type 1 diabetes mellitus may become hypoglycemic while
exercising. Someone must accompany her for her safety. The client should exercise at
the same time each day. She needs to exercise after meals, when blood glucose is
high. Fluids aren't necessary, but the client needs to bring a simple carbohydrate with
her to treat hypoglycemia.

35. A nurse is caring for a client who is scheduled for amniocentesis. What
information about the procedure should the nurse provide before the client signs the
consent form?
A. Name of procedure, how it's performed, description of alternate methods
available, potential risk to mother and fetus, risks associated if the procedure
isn't performed
B. Name of procedure, risk to mother, name of physician who will perform
procedure
C. Name of procedure, risks to the fetus
D. Description of alternate methods available, duration of the procedure, day and
time the scheduled procedure will be performed

RATIONALE: A. Before giving consent, a client has the right to be informed about
the procedure. The nurse should tell the client the name of the procedure, how it will
be performed, if alternate methods are available, the risks to the mother and fetus, and
the risk associated with an improperly performed procedure or not having the
procedure.

When determining maternal and fetal well-being, which assessment is least


important?
A. Signs of orthostatic hypotension
B. Fetal heart rate and activity
C. The mother's acceptance of the growing fetus
D. Signs of facial or digital edema

RATIONALE: A. Orthostatic hypotension doesn't occur until late in the pregnancy


and is easily correctable. Collection of other assessment data, such as fetal heart rate
and activity, the mother's acceptance of the growing fetus, and signs of edema, should
be started early in the pregnancy because abnormalities can put the mother or the fetus
at risk for significant physiological and psychological problems.

38. A client with diabetes delivers a 2,721.6-g neonate at 36 weeks' gestation. The
neonate is placed in the neonatal intensive care unit. The mother is grieving over the
early delivery. What action by the nurse should be most helpful to the client?
A. Seek involvement of external support systems to provide emotional comfort
and material resources for the client.
B. Generalize how the client must be feeling based on her written history and
pregnancy course.
C. Encourage the client to immerse herself in her intense feeling of grief.
D. Call the client's minister to obtain spiritual support for her and her family.

RATIONALE: A. Based on their knowledge of the client's unique needs and coping
mechanisms, close family and friends can offer support and resources to deal most
effectively with the client's concerns. Seeking involvement of external support
systems to provide emotional comfort and material resources for the client will help
the client express her feelings. Although expressing grief is important, it doesn't
address the critical need for support systems. Calling the client's minister to obtain
spiritual support for her and her family is one of many support systems covered by
seeking involvement of external support systems to provide emotional comfort and
material resources for the client.

A hospitalized client who is 26 weeks pregnant has been diagnosed with gestational
diabetes. She's overweight and doesn't understand the diet a physician has suggested
to her. To help the client understand the diet, the nurse should ask the physician to
order consults with a:
A. social worker.
B. dietitian.
C. psychologist.
D. lactation consultant.

RATIONALE: B Dietitian can create a meal plan that will fit into the client's lifestyle
and the specific caloric intake the physician recommends. Dietary considerations
include culture, food preferences, and activities of daily living. The nurse should
consult a social worker and a psychologist if the client's psychological needs must be
considered. The nurse should consult a lactation consultant in the postpartum stage to
address client and infant needs associated with breast-feeding.

A primigravida client is 16 weeks pregnant. Which client teaching instruction is the


most important to prevent toxoplasmosis?
A. Cook meats thoroughly.
B. Keep dogs outside.
C. Wash and cook all vegetables.
D. Have antibody titers drawn on a routine basis.

RATIONALE: A. Undercooked fresh meats that contain cysts with toxoplasmosis


can cause infection. Cats, not dogs, carry toxoplasmosis. Vegetables don't carry
toxoplasmosis. Antibody titers won't prevent
toxoplasmosis.

Which instruction should a nurse include in a home-safety teaching plan for a


pregnant client?
A. Place a nonskid mat on the floor of the tub or shower.
B. It's OK to clean your cat's litter box.
C. It's OK to wear high heels.
D. Avoid having area rugs around your house.

RATIONALE: A. Using a mat for the floor of the shower or tub will prevent slipping.
The client shouldn't clean the cat's litter box because doing so puts her at risk for
toxoplasmosis. Wearing high heels may make the client lose balance and fall. The
client doesn't need to completely avoid having area rugs around her house. Nonslip
rugs can be used to prevent tripping or falling.

A client who is in her third trimester presents at the labor and delivery triage area. She
has bruising on her back and arms and is experiencing bright red vaginal bleeding. A
completed Abuse Assessment Screen indicates the possibility of abuse. The nurse
should refer this client to:
A. the physician on call.
B. the social worker on call.
C. Women in Distress.
D. a lawyer.

RATIONALE: B. The social worker on call knows how to make a referral to


authorities without violating the client's rights. The nurse doesn't need to contact the
physician because the physician would also refer the client to the social worker. The
nurse doesn't have the right to refer the client to Women in Distress, an organization
that shelters victims of abuse, without the client's permission. It isn't appropriate for
the nurse to refer the client to a lawyer.

A 19-year-old primigravida is being treated for her second case of simple vaginitis
during pregnancy. Which instruction is most important for the nurse to focus on
during client teaching?
A. "Increase the pH of the vagina by douching regularly."
B. "Douche daily with a mild soap solution."
C. "Maintain cleanliness and avoid contamination after elimination."
D. "Report any signs and symptoms immediately."

RATIONALE: C. Simple vaginitis can result from poor hygiene, tight clothing, or
emotional stress. Teaching the client proper hygienic measures could help prevent a
recurrence. Douching isn't recommended during pregnancy- In any case, the client
should decrease vaginal pH to support the Doderlein's bacilli, the main defense of the
vagina. If douching were ordered, the pH should be lowered by using a weak acid
solution (such as vinegar and water), not a soap solution. Immediately reporting signs
and symptoms isn't going to prevent recurrence.

A primipara at 32 weeks' gestation presents complaining of vaginal bleeding. She has


soaked one peripad. She has no pain or cramps. In performing an assessment, the
nurse should suspect which condition?
A Placenta previa
(A Abruptio placentae
A Vasa previa
^ Incompetent cervix

RATIONALE: Painless vaginal bleeding is the classic sign of placenta previa.


Abruptio placentae is painful. Vasa previa occurs with ruptured membranes. An
incompetent cervix causes pressure sensations-

57. A nurse has a client at 30 weeks' gestation who has tested positive for the human
immunodeficiency virus (HIV). What should the nurse tell the client when she says
that she wants to breast-feed her neonate?
A Encourage breast-feeding so that she can get her rest and get healthier.
A Encourage breast-feeding because it's healthier for the neonate.
A Encourage breast-feeding to facilitate bonding.
A Discourage breast-feeding because HIV can be transmitted through breast milk.

RATIONALE: Transmission of HIV can occur through breast milk, so breast-feeding


should be discouraged in this case.

58. During a home visit, the client, a single multigravida at 32 weeks' gestation, tells
the nurse that she craves and often eats laundry starch for lunch and usually has a
bowl of soup for supper. Total weight gain to date has been 12 lb (5 kg). A priority
nursing diagnosis for the client is:
A Impaired parenting related to single status.
^% ineffective coping related to pregnancy.
A Imbalanced nutrition: Less than body requirements related to pica.
A Noncompliance to dietary recommendations related to insufficient resources.
RATIONALE: The priority nursing diagnosis is Imbalanced nutrition: Less than body
requirements related to pica. Pica is the term used when clients eat products that aren't
meant for consumption. The client has gained only 12 lb to date, which is below the
recommended average. No evidence exists to suggest impaired parenting, ineffective
coping, or noncompliance related to insufficient resources.

59. A pregnant client who's diabetic is at risk for having a large-for-gestational-age


infant because of which of the following?
Excess glucose causing reduced placental functioning Insulin acting as a growth
hormone on the fetus Maternal dietary intake that's high in calories Excess insulin
reducing placental functioning

RATIONALE: Insulin acts as a growth hormone on the fetus. Therefore, pregnant


diabetic clients must maintain good glucose control. Large babies are prone to
complications and may have to be delivered by cesarean section. Neither excess
glucose nor excess insulin reduces placental functioning. A high-calorie diet helps
control the mother's disease and doesn't contribute to neonatal size.

60. A nurse is caring for a 35-year-old multipara who delivered a full-term infant by
cesarean birth because of a breech presentation. The nurse recognizes that which
would be most important intervention in preventing thromboembolism?
A Increasing oral fluid intake
A Providing oxygen therapy
A Administering pain medications as needed
A Encouraging frequent ambulation

RATIONALE: Encouraging frequent ambulation would be the most important


intervention for preventing thromboembolism. Clotting factors and fibrinogen are
increased in the immediate postpartum period. When the client is in this
hypercoagulable state, the vessel damage that occurs with birth and immobility
predisposes her to developing thromboembolism. Although increasing oral fluid
intake also is important, encouraging frequent ambulation is most important.
Providing oxygen therapy and administering pain medications don't prevent
thromboembolism formation.

A primigravida at 36 weeks' gestation tells the nurse that she has moderate breast
tenderness. After providing the client with some suggestions for relief measures, the
nurse determines that the client needs further instructions when she says:
A. "I should wear a supportive bra at all times."
B. "I should clean my nipples with soap."
C. "I should change my sleeping position."
D. "I should clean up the colostrum with water."

RATIONALE: B. The client needs further instructions when she says she should
clean her nipples with soap. Soap can be extremely irritating to sensitive nipples. The
client should wear a supportive bra at all times, change her sleeping position, and
clean up the colostrum with water.

In assessing a client for pregnancy, the nurse should look for which positive sign?
A. Quickening
B. Amenorrhea
C. Fetal movement felt by the examiner
D. Enlarged uterus

RATIONALE: C. Fetal movement detected by an examiner is an objective, positive


sign of pregnancy. Quickening, a subjective sign of pregnancy experienced by the
woman, isn't as reliable as an independent assessment. Amenorrhea and enlargement
of the uterus can occur in a molar pregnancy or for various reasons unrelated to
pregnancy.

As part of a prenatal nutritional teaching program for a 17-year-old primigravida who


is concerned about weight gain, which statement by the nurse is the most accurate?
A. "If you stay away from fast foods, your weight gain will be minimal."
B. "You're young. You'll be able to lose the weight after the baby is born with no
problems."
C. "During pregnancy, a woman's caloric needs increase by about 300 calories
per day. If you like. I can help you with some meal planning."
D. "Keep your caloric intake to around 1,000 calories per day. In this way you'll
gain only the proper amount of weight."

RATIONALE: C. Helping the adolescent with a meal plan supplies her with facts and
offers help and guidance for a healthier pregnancy. Telling her to stay away from fast
foods or assuring her that she'll lose weight quickly after the birth are unrealistic and
offer false reassurance. A diet of 1,000 calories per day is insufficient to nourish an
adolescent girl and the growing fetus.

83. A nursery nurse is teaching a small-group teaching session for new parents in
preparation for discharge To comply with the law, the nurse instructs the parents that
for the automobile trip home, the neonate should be in an approved car seat in the:
A. back seat facing the front.
B. front seat facing the back.
C. back seat facing the back.
D. front seat facing the front.

RATIONALE: C. Neonates (all infants) up to 20 lb should be placed in an approved


car seat in the back seat facing the back. This position provides the most protection
for the infant in the event of an accident. Infants facing the front might be thrown
forward in an accident. Infants in the front seat are at greater risk for injury during an
accident.

84. A client who is 24 weeks pregnant states that her employer has changed health
insurance companies. She's worried that she won't be able to continue to receive
prenatal care at her current facility. A nurse informs her that because she's an
established client at the facility, she'll continue to receive care through delivery. This
is referred to as:
A. continuity of care.
B. freedom of choice.
C. informed consent.
D. the Consolidated Omnibus Budget Reconciliation Act (COBRA)
RATIONALE: A. Continuity of care allows a client to continue receiving care from
an established provider until she's discharged from care. The new health insurance
company must honor the cost. This situation isn't an example of freedom of choice
because the employee doesn't have a say in choosing an employer's insurance
company or electing to keep the original company. Informed consent involves
obtaining permission from a client to perform a specific test or procedure after the
client has been fully informed about the test or procedure. COBRA is a program that
allows eligible former employees to continue their health care coverage after leaving
an employer.

85. A nurse is caring for a client in the first 4 weeks of pregnancy. The nurse should
expect to collect which assessment findings?
A. Presence of menses
B. Uterine enlargement
C. Breast sensitivity
D. Fetal heart tones

RATIONALE: C, Breast sensitivity is the only sign assessed within the first 4 weeks
of pregnancy. Amenorrhea is expected during this time. The other assessment
findings don't occur until after the first 4 weeks of pregnancy.

. A client diagnosed with gestational hypertension must have weekly blood pressure
checks and urine testing at a clinic. She doesn't have transportation. How can the
nurse help this client be compliant with her care?
A. Ask the clinic case manager to speak with the client.
B. Set up cab service.
C. Do nothing. It's the client's responsibility to find a way to get to the clinic.
D. Ask the client to find a friend to help her.

RATIONALE: A. The nurse should ask the case manager to speak with the client
because the case manager is familiar with community resources that can assist with
transportation. Resources and additional support will greatly increase the client's
compliance. The nurse can't set up cab service if the client doesn't have the funds to
pay for transportation. The client may be noncompliant if she has no assistance or if
she has to rely on a friend to help.

. A nurse is teaching a group of couples in a childbirth class. After the nurse describes
normal labor, including the premonitory signs of labor, the clients attending the class
comment. Which remark would indicate that further teaching is necessary?
A. "My membranes won't rupture until I'm ready to deliver."
B. "I may feel Braxton Hicks contractions as my pregnancy progresses."
C. "Lightening usually occurs 2 weeks before labor begins in a first pregnancy."
D. "I'll begin to see a bloody mucus vaginal discharge as my cervix begins to
dilate."

RATIONALE: Class member saying her membranes won't rupture until she's ready
to deliver is an indication that further teaching is necessary. Most clients' membranes
will rupture before the time of delivery, but approximately 12% will rupture before
the onset of labor (known as premature rupture of membranes [PROM]). Premonitory
signs of labor include Braxton Hicks contractions; lightening (which usually occurs 2
weeks before labor begins in a first pregnancy); cervical changes, including softening
and ripening; bloody mucus vaginal discharge (bloody show); rupture of the
membranes; a sudden burst of energy; weight loss; increased backache; and diarrhea,
indigestion, nausea, or vomiting.

During routine prenatal screening, a nurse tells a client that her blood sample will be
used for alpha fetoprotein (AFP) testing. Which statement best describes what AFP
testing indicates?
A. "This test will show if you have gestational diabetes."
B. "This test will screen for spina bifida, Down syndrome, or other genetic
defects."
C. "This screening indicates if your baby's lungs are mature."
D. 'To provide accurate results, this screening must be performed exactly at 25
weeks' gestation."

RATIONALE: B. AFP testing screens for spina bifida, Down syndrome, and other
genetic defects. It must be performed at 16 to 18 weeks' gestation to provide accurate
results. A 1-hour glucose tolerance test diagnoses gestational diabetes. Amniocentesis
assesses the maturity of fetal lungs.

A nurse is providing care for a pregnant client. The client asks the nurse how she can
best deal with her fatigue. The nurse should instruct her to:
A. take sleeping pills for a restful night's sleep,
B. try to get more rest by going to bed earlier.
C. take her prenatal vitamins.
D. not worry because the fatigue will go away soon.

RATIONALE: B. The client should listen to the body's way of telling her that she
needs more rest and try going to bed earlier. Sleeping pills shouldn't be consumed
prenatally because they can harm the fetus. Vitamins won't take away fatigue. False
reassurance is inappropriate and doesn't help her deal with fatigue now.

A nurse is assessing a pregnant woman in the clinic. In the course of the assessment,
the nurse learns that this woman smokes one pack of cigarettes per day. The first step
the nurse should take to help the woman stop smoking is to:
ft assess the client's readiness to stop.
ft suggest that the client reduce the daily number of cigarettes smoked by one-half.
ft provide the client with the telephone number of a formal smoking cessation
program.
£ help the client develop a plan to stop.

RATIONALE: A. Before planning any intervention with a client who smokes, it's
essential to determine whether the client is willing or ready to stop smoking.
Commonly, a pregnant woman will agree to stop smoking for the duration of the
pregnancy. This temporary cessation gives the nurse an opportunity to work with the
client over time to help with permanent smoking cessation.

A client who's planning a pregnancy asks the nurse about ways to promote a healthy
pregnancy. Which response by the nurse would be best?
A. "Pregnancy is a human process; you don't have to worry."
B. "You practice good health habits; just follow them and you'll be fine."
C. "There's nothing you can do to have a healthy pregnancy; it's all up to nature."
D. "Folic acid, 400 meg, improves pregnancy outcomes by preventing certain
complications."

RATIONALE: D. When counseling a client who's planning to become pregnant, the


nurse should discuss the role of folic acid in preventing neural tube defects. The nurse
should provide information but not prescribe the drug. It's the client's responsibility to
ask the health care provider about a prescription. Telling the client not to worry
ignores the client's needs. Practicing good health habits is important for any person.
Telling the client that it's up to nature is inaccurate.

A nurse is caring for a neonate of a client with type 1 diabetes mellitus. The neonate
at high risk for hypoglycemia. An initial sign in a neonate that indicates hypoglycemia
is:
A. peripheral acrocyanosis
B. bradycardia.
C. jitteriness.
D. jaundice.

RATIONALE: C. Jitteriness in the neonate may be seen with hypoglycemia because


of a lack of glucose in the nerve cells. Peripheral acrocyanosis is normal in the
neonate because of immature capillary function. Tachycardia, not bradycardia, is seen
with hypoglycemia. Jaundice isn't a sign of hypoglycemia.

A primipara client is 6 weeks pregnant. At her first prenatal visit, she says to the
nurse, "I just can't believe I'm really pregnant. I hope this baby is a good idea." What
should the nurse make of the client's statement?
A. The client is afraid of pregnancy and birth.
B. The client should have waited until she was committed to having a baby.
C. The client is experiencing normal ambivalence about being pregnant.
D. The client may have problems attaching to the baby after birth.

RATIONALE: C. Ambivalence is normal in the first trimester of pregnancy, even


when a pregnancy is planned and desired. Fear and attachment problems aren't
substantiated by the evidence supplied. Assuming that the client should have waited
to have her baby is judgmental.

42. A nurse is doing an assessment of a pregnant client who states that she smokes
one pack of cigarettes each day. The nurse should:
A. ask the client to cut down to half a pack a day.
B. inform the client of the risks to the fetus and ask if she'd like a referral to a
smoking cessation support group.
C. insist that the client stop smoking immediately for the health of her baby,
D. do nothing; smoking is a personal decision.

RATIONALE: B. Smoking can be detrimental to fetal development and cause low


birth weight and preterm birth. Informing the client of the risk factors and offering to
help with a referral to a support group provides encouragement and respect for the
client's right to make the decision. Reducing the number of cigarettes won't lessen the
risks to the fetus. Insisting that the client stop smoking could result in a breakdown of
established trust. Not taking any action results in not providing adequate health care
information.

43. Which of the following describes the term fetal position?


A. Relationship of the fetus's presenting part to the mother's pelvis
B. Fetal posture
C. Fetal head or breech at cervical os
D. Relationship of the fetal long axis to the mother's long axis

RATIONALE: A. Fetal position refers to the relationship of the fetus's presenting part
to the mother's pelvis. Fetal posture refers to "attitude." Presentation refers to the part
of the fetus at the cervical os. Lie refers to the relationship of the fetal long axis to that
of the mother's long axis.

44. A nurse is caring for a client in the first 4 weeks of pregnancy. The nurse should
expect to collect which assessment findings?
A. Presence of menses
B. Uterine enlargement
C. Breast sensitivity
D. Fetal heart tones

RATIONALE: C. Breast sensitivity is the only sign assessed within the first 4 weeks
of pregnancy. Amenorrhea is expected during this time. The other assessment
findings don't occur until after the first 4 weeks of pregnancy.

. Which of the following would the nurse expect to assess as presumptive signs of
pregnancy?
A. Amenorrhea and quickening
B. Uterine enlargement and Chadwick's sign
C. positive pregnancy test and a fetal outline
D. Braxton Hicks contractions and Hegar's sign

RATIONALE: A. Presumptive signs, such as amenorrhea and quickening, are mostly


subjective and may be indicative of other conditions or illnesses. Probable signs are
objective but nonconclusive indicators — for example, uterine enlargement,
Chadwick's sign, a positive pregnancy test, Braxton Hicks contractions, and Hegar's
sign. Positive signs and objective indicators such as fetal outline on ultrasound
confirm pregnancy.

As she tries to decide on a birth-control method, a client requests information about


medroxyprogesterone (Depo-Provera). Which statement represents the nurse's best
response?
A. "Depo-Provera needs to be administered every 12 weeks."
B. "Depo-Provera is effective for only 2 months at a time."
C. "Depo-Provera can't be given to breast-feeding women."
D. "Depo-Provera has a high failure rate; use a barrier form of protection also."
RATIONALE: A. Depo-Provera will provide effective birth control for 3 months, and
it may be the birth-control method of choice for clients who are breast-feeding
because studies haven't established any contraindications. There's no evidence that the
drug has a high failure rate.

Which hormone is responsible for the let-down reflex?


A. Oxytocin
B. Prolactin
C. Estrogen
D. Progesterone

RATIONALE: A. Oxytocin is responsible for milk let-down, the process that brings
milk to the nipple. The other hormones mentioned contribute indirectly to the
lactation process. Prolactin stimulates lactation. Estrogen stimulates development of
the duct in the breast. Progesterone acts to increase the lobes, lobules, and alveoli of
the breasts.

57. When determining maternal and fetal well-being, which assessment is least
important?
A. Signs of orthostatic hypotension
B. Fetal heart rate and activity
C. The mother's acceptance of the growing fetus
D. Signs of facial or digital edema

RATIONALE: A. Orthostatic hypotension doesn't occur until late in the pregnancy


and is easily correctable. Collection of other assessment data, such as fetal heart rate
and activity, the mother's acceptance of the growing fetus, and signs of edema, should
be started early in the pregnancy because abnormalities can put the mother or the fetus
at risk for significant physiological and psychological problems.

58. A nurse is caring for a primigravida who's scheduled for a fetal acoustic
stimulation test (FAST). The nurse should explain to the client that the primary
purpose of this test is to:
A. induce contractions.
B. induce fetal heart rate accelerations.
C. shorten the contraction stress test,
D. determine fluid volume.

RATIONALE: D. The FAST is a noninvasive technique that induces fetal heart rate
accelerations by using low-frequency vibrations on the maternal abdomen over the
fetal head. It can shorten the length of the nonstress test. The FAST isn't used to
induce contractions, shorten the length of the contraction stress test, or determine fluid
volume.

59. After determining that a pregnant client is Rh-negative, the physician orders an
indirect Coombs' test. What's the purpose of performing this test on a pregnant client?
A. To determine the fetal blood Rh factor
B. To determine the maternal blood Rh factor
C. To detect maternal antibodies against fetal Rh-negative factor
D. To detect maternal antibodies against fetal Rh-positive factor
RATIONALE: D. The indirect Coombs' test measures the level of antibodies against
fetal Rh-positive factor in maternal blood. Although this test may determine the fetal
blood Rh factor, the physician doesn't order it primarily for this purpose. The maternal
blood Rh factor is determined before the indirect Coombs' test is done. No maternal
antibodies against fetal Rh-negative factor exist.

60. A client is having a level 2 ultrasound. A nurse knows that physicians order this
procedure:
A. to satisfy the client's curiosity.
B. to assess the correct date of gestation.
C. for diagnostic purposes when fetal development is in question.
D. to provide images of the fetus for family and friends.

RATIONALE: C. Level 2 ultrasound is more sophisticated and can visualize fetal


structures more clearly than a level 1 ultrasound. It's used for diagnostic purposes
when fetal development is in question. Typically, level 1 ultrasound is used to assess
gestational age. Diagnostic ultrasounds aren't ordered to satisfy the client's curiosity
or to provide images of the fetus for family and friends.

When assessing a client during her first prenatal visit, the nurse discovers that the
client had a reduction mammoplasty. The mother indicates that she wants to breast-
feed. What information should the nurse give to this mother regarding breast-feeding
success?
A. "It's contraindicated for you to breast-feed following this type of surgery”
B. "I support your commitment; however, you may have to supplement each
feeding with formula."
C. "You should check with your surgeon to determine whether breast-feeding
would be possible."
D. "You should be able to breast-feed without difficulty."

RATIONALE: B. Breast reduction surgeries are currently done in a way that protects
the milk sacs and ducts, so breast-feeding after surgery is possible. Still, it's good to
check with the surgeon to determine what breast reduction procedure was done. There
is the possibility that reduction surgery may have decreased the mother's ability to
meet all of her baby's nutritional needs, and some supplemental feeding may be
required. Preparing the mother for this possibility is extremely important because the
client's psychological adaptation to mothering may be dependent on how successfully
she breast-feeds. Telling the client she should be able to breast-feed without difficulty
is inappropriate because supplemental feedings may be needed.

66. A client who has received a new prescription for oral contraceptives asks the
nurse how to take them.
Which of the following would the nurse instruct the client to report to her primary
caregiver?
A. Breast tenderness
B. Breakthrough bleeding within first 3 months of use
C. Decreased menstrual flow
D. Blurred vision and headache
RATIONALE: D. Some adverse effects of birth control pills, such as blurred vision
and headaches, require a report to the health care provider. Because these two effects
in particular may be precursors to cardiovascular compromise and embolus, the client
may need to use another form of birth control. Breast tenderness, breakthrough
bleeding, and decreased menstrual flow may occur as a normal response to the
use of birth control pills.

A nurse is teaching a new mother about exercise and injury prevention. Which
statement by the client requires further teaching?
A. "I can perform muscle flexing and stretching."
B. "I should do Kegel exercises."
C. "Pelvic tilts will help my back." I
D. “I’m allowed to jog in place."

RATIONALE: D. The client requires additional teaching if she states that she may jog
in place. Jogging can increase the amount of lochia rubra, which indicates new
bleeding. Muscle flexing and stretching, Kegel exercises, and pelvic tilts are safe to
do during the first 3 weeks postpartum. Stretching and flexing muscles relieves
tension. Kegel exercises tone the muscles of the pelvic floor. Pelvic tilts strengthen
the muscles of the lower back.

. A 19-year-old primigravida tells the nurse that the physician told her that she needed
to increase her intake of thiamine (vitamin B-j) in her diet. The nurse should instruct
the client to consume more:
A. milk.
B. rice.
C. asparagus.
D. beef.

RATIONALE: A. Good sources of thiamin include milk, pork, liver, potatoes,


enriched cereals, and enriched breads. Rice, asparagus, and beef aren't good sources
of thiamine.

73. A client asks about complementary therapies for relief of discomforts related to
pregnancy. Which comfort measure mentioned by the client indicates a need for
further teaching?
A. Meditation
B. Music therapy
C. Acupuncture
D. Herbal remedies

RATIONALE: D. Pregnant woman should avoid all medication unless her physician
instructs her to use it. This includes herbal remedies because their effects on the fetus
haven't been identified. Meditation, music therapy, and acupuncture have all proven
to enhance relaxation without harm to the mother or baby.

A woman who is 10 weeks pregnant tells the nurse that she's worried about her
fatigue and frequent urination. The nurse should:
A. assure the client that these are normal early pregnancy signs and symptoms,
B. question her further about these signs and symptoms, t
C. ell her that she'll need blood work and urinalysis.
D. ell her that she may be excessively worried.

RATIONALE: A. Fatigue and frequent urination are early signs and symptoms of
pregnancy that may continue through the first trimester. The nurse should assure the
client that these signs and symptoms are normal. Questioning the client about the
signs and symptoms is helpful to complete the assessment but won't reassure her.
Prenatal blood work and urinalysis is routine for this situation but doesn't address the
client's concerns. Telling her that she may be excessively worried isn't therapeutic.

A client in the 13th week of pregnancy develops hyperemesis gravidarum. Which


laboratory finding indicates the need for intervention?
A. Urine specific gravity 1.010
B. Serum potassium 4 mEq/L
C. Serum sodium 140 mEq/L
D. Ketones in urine

RATIONALE: D. Ketones in the urine of a client with hyperemesis gravidarum


indicate that the body is breaking down stores of fat and protein to provide for growth
needs. Urine specific gravity of 1.010, serum potassium of 4 mEq/L, and serum
sodium of 140 mEq/L listed are within normal limits.

80. A nurse should anticipate which psychological reactions in a client in her second
trimester of pregnancy?
A. Self-centeredness and concentration on the behavior and appearance of
children
B. Extroversion and emotional lability
C. Ambivalence and uncertainty
D. Dismay over body image and readiness for the end of pregnancy

RATIONALE: A. Women during the second trimester are somewhat narcissistic; at


the same time, they're commonly fascinated by children. Extroversion is a personality
trait not specific to pregnancy; emotional lability may be present in every trimester.
Ambivalence and uncertainty are characteristic of the first trimester. Dismay over
body image and readiness for pregnancy to be over are characteristic of the third
trimester.

Which finding would be an indication of placental detachment?


A. An abrupt shortening of the cord
B. An increase in the number of contractions
C. Relaxation of the uterus
D. Decreased vaginal bleeding

RATIONALE: B. An increase in the number of contractions, an abrupt lengthening


(not shortening) of the cord, and an increase (not decrease) in vaginal bleeding are all
indications that the placenta has detached from the wall of the uterus. Relaxation isn't
an indication of detachment of the placenta.

82. A mother is concerned that her neonate son, who was delivered without
complications at 38 weeks, isn't eating enough and will lose too much weight. The
mother states, "He only breast-feeds for about 3 minutes on one side." Which
instructions should the nurse provide to this mother?
A. "Don't worry. When he's hungry, he'll eat. You'll see; it will be fine."
B. "I understand your concern. He has stored nutrients before birth just for this
reason."
C. "It's important that he doesn't lose too much weight. We should start him on
formula after each feeding."
D. "I'm concerned, too, and will notify the pediatrician immediately."

RATIONALE: B. Neonates who are born at term without complications have stores
of brown fat located on the vital organs. These stores will provide the infant with the
needed calories until lactation is well established. Cold, stress, and transitional
neonatal problems may use up the stores of brown fat. Telling the client not to worry
and saying things will be fine ignores the mother's concerns. Starting the neonate on
formula and notifying the physician are inappropriate at this time.

83. Lower back pain is a common complaint among pregnant clients. Which comfort
measure should a nurse include in her teaching plan for a pregnant client?
A. Wear high-heeled shoes.
B. Use an economically correct desk chair.
C. Avoid tilting the pelvis forward.
D. Bend at the waist, not at the knees.

RATIONALE: C. The nurse should instruct the client to use an economically correct
desk chair to help alleviate lower back pain. Wearing high heels promotes imbalance
and falls. The nurse shouldn't instruct the client to avoid tilting the pelvis forward
because standing with her neck and shoulders straight and pelvis tilted forward
alleviates stress caused by excess uterine weight. Bending and lifting at the knees (not
at the waist) alleviates strain on lower-back muscles.

84. A newly pregnant woman tells the nurse that she hasn't been taking her prenatal
vitamins because the} make her nauseous. In addition to stressing the importance of
taking the vitamins, the nurse should
advise the client to:
A. switch brands.
B. take the vitamin on a full stomach.
C. take the vitamin with orange juice for better absorption.
D. take the vitamin first thing in the morning.

RATIONALE: B. Prenatal vitamins commonly cause nausea and taking them on a


full stomach may curb this effect. Switching brands may not be helpful and may be
more costly. Orange juice tends to make pregnant women nauseous. Prenatal vitamins
may be taken at night, rather than in the morning, to reduce nausea.

85. A pale, thin 15-year-old adolescent comes to the clinic for a pregnancy test, which
is positive. The adolescent states that she and her boyfriend have run away from home
and that both are unemployed Which is the most therapeutic statement to begin
assessment and establish a relationship based on trust?
A. "How does your mother feel about your situation?"
B. "Have you talked to your mother?"
C. "Sometimes young women find it hard to talk about this with their mothers."
D. "You don't have to tell your mother. What are you going to do?"

RATIONALE: C. This statement confirming the possible difficulty in discussing a


pregnancy with her mother may encourage the adolescent to explore her feelings with
a nurse who seems to understand. Asking how the adolescent's mother feels about the
pregnancy or telling her that she doesn't have to tell her mother might elicit some
information, but they make assumptions about the adolescent's situation in a way that
seems likely to limit further communication or trust. Asking the adolescent if she has
spoken with her mother is a closed question. Even if the response is "yes," the
adolescent may have given the answer she thinks the nurse wants to hear rather than
telling the truth.

During the first 3 months, which hormone is responsible for maintaining pregnancy?
A. Human chorionic gonadotropin (HCG)
B. Progesterone
C. Estrogen
D. Relaxin

RATIONALE: A. HCG is the hormone responsible for maintaining the pregnancy


until the placenta is in place and functioning. Serial HCG levels are used to determine
the status of the pregnancy in clients with complications. Progesterone and estrogen
are important hormones responsible for many of the body's changes during
pregnancy. Relaxin is an ovarian hormone that causes the mother to feel tired, thus
promoting her to seek rest.

Assessment of a pregnant client reveals that she feels very anxious because of a lack
of knowledge about giving birth. The client is in her second trimester. Which
intervention by the nurse is most appropriate for this client?
A. Provide her with the information and teach her the skills she'll need to
understand and cope during birth.
B. Provide her with written information about the birthing process.
C. Have a more experienced pregnant woman assist her.
D. Do nothing in hopes that she'll begin coping as the pregnancy progresses.

RATIONALE: A. Because the client is in her second trimester, the nurse has ample
time to establish a trusting relationship with her, provide her with information, and
teach her coping skills in a style that fits her needs. Written information would be
effective only in conjunction with teaching sessions. Introducing her to another
pregnant client may be helpful, but the nurse still needs to teach the client about
giving birth. Doing nothing won't address the client's needs.

What's the approximate time that the blastocyst spends traveling to the uterus for
implantation?
A. 2 days
B. 7 days
C. 10 days
D. 14 weeks
RATIONALE: B. The blastocyst takes approximately 1 week to travel to the uterus
for implantation. The other options are incorrect.

A. 4. A client, now 37 weeks pregnant, calls the clinic because she's concerned
about being short of breath and is unable to sleep unless she places three
pillows under her head. After listening to her concerns, the nurse should take
which action?
B. Make an appointment because the client needs to be evaluated.
C. Explain that these are expected problems for the latter stages of pregnancy.
D. Arrange for the client to be admitted to the birth center for delivery.
E. Tell the client to go to the hospital; she may be experiencing signs of heart
failure from a 45% to 50% increase in blood volume.

RATIONALE: B. The nurse must distinguish between normal physiological


complaints of the latter stages of pregnancy and those that need referral to the health
care provider. In this case, the client indicates normal
physiological changes due to the growing uterus and pressure on the diaphragm. The
client doesn't need to be seen or admitted for delivery. The client's signs aren't
indicative of heart failure.

A nurse is assessing a pregnant woman in the clinic. In the course of the assessment,
the nurse learns that this woman smokes one pack of cigarettes per day. The first step
the nurse should take to help the woman stop smoking is to:
A. assess the client's readiness to stop.
B. suggest that the client reduce the daily number of cigarettes smoked by one-
half.
C. provide the client with the telephone number of a formal smoking cessation
program.
D. help the client develop a plan to stop.

RATIONALE: A. Before planning any intervention with a client who smokes, it's
essential to determine whether the client is willing or ready to stop smoking.
Commonly, a pregnant woman will agree to stop smoking for the duration of the
pregnancy. This temporary cessation gives the nurse an opportunity to work with the
client over time to help with permanent smoking cessation.

8. At 32 weeks' gestation, a client is admitted to the hospital with a diagnosis of


gestational hypertension. Based on this diagnosis, the nurse expects assessment to
reveal which sign?
A. Edema
B. Fever
C. Glycosuria
D. Vomiting

RATIONALE: A. Classic signs of gestational hypertension include edema (especially


of the face), elevated blood pressure, and proteinuria. Fever is a sign of infection.
Glycosuria indicates hyperglycemia. Vomiting may be associated with any number of
disorders.
A nurse is caring for a 15-year-old primigravida who visits the clinic for the first time
at 20 weeks' gestation. A priority goal for this client is that she'll be able to:
A. maintain a steady weight gain until term.
B. record the number of fetal movements four times daily.
C. attend prenatal care appointments on a regular basis.
D. explain the process of fetal development.

RATIONALE: C. The purpose of prenatal care is to detect signs and symptoms of


complications early so treatment can begin and fetal outcome will be good. This
adolescent client visits the clinic for the first time at 20 weeks' gestation; therefore, a
priority goal is that the client attends prenatal appointments on a regular basis.
Maintaining a steady weight gain isn't a priority goal unless there's some indication
that the client hasn't been maintaining a steady and appropriate weight gain.
Recording the number of fetal movements four times daily isn't indicated, and
explaining the process of fetal development isn't a priority at 20 weeks' gestation.

12. A nurse is reviewing prenatal care with a client. Which statement by the client
best expresses adequate understanding of nutritional needs during pregnancy?
A. "I expect to gain a few pounds each month at first. Then I'll really get big and
put on 20 pounds or so."
B. "I guess I will get big and gain 20 to 30 pounds and look pregnant."
C. "Because I have to eat for two, I should eat whatever I want whenever I feel
hungry."
D. "I will need to eat more so that I will gain about 25 pounds, but I want to make
sure I don't fill up with junk food."

RATIONALE: D. Stating that a 25-pound (11.3 kg) weight gain is expected with
avoidance of junk food shows that the client has an understanding of nutritional needs
during pregnancy. A slow weight gain followed by a larger gain is accurate but
doesn't express an understanding of nutritional needs. A gain of20 to 30 pounds (9 to
13.5 kg) and "looking pregnant" doesn't show an understanding of either nutritional
needs or how and when the weight gain will occur. "Eating for two" is a common
rationalization that can result in excessive weight gain.

13. A client with type 1 diabetes mellitus is gravida 2, para 0, abortus 1. She's now at
37 weeks' gestation. Several tests to determine fetal lung maturity have been
performed by amniocentesis. Which is the most reliable indicator of fetal lung
maturity?
A. Lecithin-sphingomyelin (US) ratio of 2:1
B. The presence of phosphatidylglycerol
C. Increasing bilirubin levels
D. Decreasing estriol levels

RATIONALE: B. The presence of phosphatidylglycerol is a reliable sign that the


lungs are mature. The L/S ratio must be higher in the diabetic because high insulin
levels inhibit surfactant production. Bilirubin levels don't measure fetal lung maturity
and should decrease at term; increasing levels may signal a fetal blood
incompatibility. Estriol levels don't measure fetal lung maturity.
A pregnant client complains of nausea every morning and again before meals. As a
result of the nausea, she's been unable to eat enough and has lost weight. Which
nonpharmacologic intervention should the nurse recommend?
A. Drinking water with every meal
B. Keeping crackers at the bedside to eat before getting out of bed
C. Eating three large meals per day
D. Drinking liquids with dry foods

RATIONALE: B. The nurse should advise the client to keep crackers at the bedside
because eating dry crackers before getting out of bed and before the stomach becomes
empty helps prevent nausea. Drinking water with every meal doesn't alleviate nausea.
Eating six small meals per day rather than three large meals prevents nausea by
preventing the stomach from becoming empty. Drinking liquids with dry food
increases nausea. The client should be instructed to wait at least 30 minutes to
consume liquids after eating dry food.

When teaching a group of pregnant teens about reproduction and conception, the
nurse is correct when stating that fertilization occurs:
A. in the uterus.
B. when the ovum is released.
C. near the fimbriated end.
D. in the first third of the fallopian tube.

RATIONALE: D. Fertilization occurs in the first third of the fallopian tube. After
ovulation, an ovum is released by the ovary into the abdominopelvic cavity. It enters
the fallopian tube at the fimbriated end and moves through the tube on the way to the
uterus. Sperm cells "swim up" the tube and meet the ovum in the first (third of the
fallopian tube. The fertilized ovum then travels to the uterus and implants. Nurses
must know where fertilization occurs because of the risk of an ectopic pregnancy.

19. A nurse is teaching a group of couples in a childbirth class. After the nurse
describes normal labor, including the premonitory signs of labor, the clients attending
the class comment. Which remark would indicate that further teaching is necessary?
A. "My membranes won't rupture until I'm ready to deliver."
B. "I may feel Braxton Hicks contractions as my pregnancy progresses."
C. "Lightening usually occurs 2 weeks before labor begins in a first pregnancy."
D. "I'll begin to see a bloody mucus vaginal discharge as my cervix begins to
dilate."

RATIONALE: A. Class member saying her membranes won't rupture until she's
ready to deliver is an indication that further teaching is necessary. Most clients'
membranes will rupture before the time of delivery, but approximately 12% will
rupture before the onset of labor (known as premature rupture of membranes
[PROM]). Premonitory signs of labor include Braxton Hicks contractions; lightening
(which usually occurs 2 weeks before labor begins in a first pregnancy); cervical
changes, including softening and ripening; bloody mucus vaginal discharge (bloody
show); rupture of the membranes; a sudden burst of energy; weight loss; increased
backache; and diarrhea, indigestion, nausea, or vomiting.
The physician placed a direct fetal scalp electrode on a fetus. What should the nurse
include when documenting direct fetal scalp electrode placement?
A. Time of fetal scalp electrode placement, name of the physician who placed the
electrode, and frequency of uterine contractions.
B. Time of fetal scalp electrode placement, name of the physician or nurse
practitioner who applied the electrode, and the fetal heart rate (FHR).
C. The name of the physician or nurse practitioner who applied the electrode,
Doppler transducer placement, and FHR.
D. The maternal and fetal body movements identified by the direct fetal scalp
electrode, time of fetal scalp electrode placement, and FHR.

RATIONALE: B. Direct fetal scalp electrode placement is the most accurate way to
assess FHR. Documentation should include the time the electrode was placed, the
name of the physician or nurse practitioner who performed the procedure, and the
FHR. Direct fetal scalp electrodes don't monitor maternal uterine contractions. A
Doppler transducer (an external, not internal, device) is applied to the mother's
abdomen to measure FHR using high-frequency ultrasound; unlike the fetal scalp
electrode, it doesn't directly measure FHR. The fetal scalp electrode doesn't measure
maternal or fetal movements.

23. A client who's 7 months pregnant reports severe leg cramps at night. Which
nursing action would be most effective in helping her cope with these cramps?
A. Suggesting that she walk for 1 hour twice per day
B. Advising her to take over-the-counter calcium supplements twice per day
C. Teaching her to dorsiflex her foot during the cramp
D. Instructing her to increase milk and cheese intake to 8 to 10 servings per day

RATIONALE: C. Common during late pregnancy, leg cramps cause shortening of the
gastrocnemius muscle in the calf. Dorsiflexing or standing on the affected leg extends
that muscle and relieves the cramp. Although moderate exercise promotes circulation,
walking 2 hours per day is excessive during the third trimester. Excessive calcium
intake may cause hypercalcemia, promoting leg cramps; the physician must evaluate
the client's need for calcium supplements. If the client eats a balanced diet, calcium
supplements or additional servings of high-calcium foods may be unnecessary.

24. A client, a gravida 3 para 2 at 35 weeks' gestation, comes to the antepartum clinic
for a check-up. She has been experiencing backaches after standing all day at her job
as a grocery clerk. The nurse should suggest to the client that she practice an exercise
such as:
A. the pelvic tilt.
B. squatting.
C. stretching.
D. walking.

RATIONALE: A. An exercise such as the pelvic tilt can help restore body alignment
and alleviate backache. Squatting strengthens the pelvic muscles. Stretching and
walking are good exercises but often don't relieve backache.

A pregnant client is taking folic acid. During prenatal teaching, the nurse should
recommend which food as high in folic acid?
A. Egg yolks
B. Fruit
C. Bread
D. Milk

RATIONALE: A. Egg yolks, nuts, seeds, and liver are all high in folic acid. Fruit,
bread, and milk aren't good sources of folic acid.

A nurse is caring for a client who is scheduled for amniocentesis. What information
about the procedure should the nurse provide before the client signs the consent form?
A. Name of procedure, how it's performed, description of alternate methods
available, potential risk to mother and fetus, risks associated if the procedure
isn't performed
B. Name of procedure, risk to mother, name of physician who will perform
procedure
C. Name of procedure, risks to the fetus
D. Description of alternate methods available, duration of the procedure, day and
time the scheduled procedure will be performed

RATIONALE: A. Before giving consent, a client has the right to be informed about
the procedure. The nurse should tell the client the name of the procedure, how it will
be performed, if alternate methods are available, the risks to the mother and fetus, and
the risk associated with an improperly performed procedure or not having the
procedure.

. A client in her seventh month of pregnancy has been complaining of back pain and
wants to know what can be done to relieve it. Which response by the nurse is most
effective?
A. "You need to lie down more during the day to get off your feet."
B. "Avoid lifting heavy loads, and try using the pelvic tilt exercise."
C. "Have others pick things up for you so you don't have to bend over so much."
D. "Your back pain will go away after the baby is born."

RATIONALE: B. The pelvic tilt exercise, which can be done standing as well as lying
down, and avoiding lifting heavy loads can greatly relieve back discomfort- As the
pregnancy progresses into the last trimester, women typically develop a "swayback"
curvature of the spine to counterbalance the enlarging fetus. Tilting of the pelvis
aligns the spine, decreasing pressure and back discomfort. Lying down more during
the day may not be possible or convenient for some clients. Also, the supine position
may not be comfortable for some clients and may cause vena cava syndrome
(dizziness on rising and decreased circulation to the fetus). Avoiding bending over as
much may not be realistic for the client's circumstances, nor does it address back pain
as effectively as the pelvic tilt. Telling the client that the pain will go away after the
baby is born doesn't help to relieve the client's discomfort.

38. A nurse is orienting a new nurse to the labor and delivery unit. Which action by
the new nurse regarding a neonate's security requires intervention by the preceptor?
A. Affixing matching identification bands to the parents and neonate at birth
B. Positioning a rooming-in neonate's bassinet toward the center of room rather
than near the door to the hallway
C. Affixing a security bracelet that monitors movement to a neonate
D. Allowing volunteers to return neonates to the nursery

RATIONALE: D. The new nurse requires additional teaching if she allows volunteers
to return neonates to the nursery. Unit staff members won't likely recognize
volunteers, whose assignments vary with each shift. Affixing matching identification
bands at birth, positioning a rooming-in neonate's bassinet toward the center of the
room, and affixing security bracelets are appropriate security measures.

39. A multigravida tells the nurse that her husband has been experiencing nausea and
vomiting and backaches that are similar to the client's discomforts during this
pregnancy. The nurse should explain to the client that this is termed:
A. neurosis.
B. psychosis
C. mimicry.
D. couvade.

RATIONALE: D. Partner who's very involved in the pregnancy occasionally


experiences discomforts similar to those of pregnancy. This is termed "couvade." The
partner isn't exhibiting neurosis, psychosis, or mimicry.

40. A client tells a nurse that she's in a nontraditional same-sex relationship. The
woman's partner is the health care surrogate for the client and her fetus. The sperm
donor, who is their best friend, has waived parental rights. If the client can't make
health care decisions for the fetus, who's responsible for making them?
A. The client's partner
B. The client's best friend, who's the sperm donor
C. The client's parents, because they're blood relatives
D. The court system, because the client isn't married and is legally responsible for
the neonate

RATIONALE: A. Legal document stating that the client's partner is the health care
surrogate for the client and the fetus authorizes the partner to make decisions on
behalf of the client or the fetus if the client isn't able to do so. Before insemination, a
donor signs a legal document waiving rights to the child; therefore, the donor has no
authority to make health care decisions on behalf of the client or the fetus. Pregnancy
at any age results in emancipation; parents don't have rights to make health care
decisions for pregnant adolescents. The court system wouldn't make the decision if
the client has designated a legal health care surrogate.

41. Infertility in a 25-year-old couple is defined as which of the following?


A. The couple's inability to conceive after 6 months of unprotected attempts
B. The couple's inability to sustain a pregnancy
C. The couple's inability to conceive after 1 year of unprotected attempts
D. low sperm count and decreased motility

RATIONALE: C. The determination of infertility is based on age. In a couple


younger than 30 years old, infertility is defined as failure to conceive after 1 year of
unprotected intercourse. In a couple age 30 or older, the time period is reduced to 6
months of unprotected intercourse.
42. A primipara client in her 10th week of pregnancy calls the nurse to say that she's
experiencing slight vaginal bleeding. What's the nurse's best response?
A. "Lie down on your left side and call again if the bleeding worsens."
B. "Save any perineal pads, clots, and tissue and come to the clinic right away."
C. "Avoid sexual intercourse for the next 2 weeks."
D. "Continue your normal activities and increase your fluid intake."

RATIONALE: B. Vaginal bleeding, a sign of threatened abortion, warrants


immediate attention. Saving perineal pads and any matter passed vaginally will make
evaluation more reliable. The left lateral position is important during the last
trimester, when the possibility of vena cava syndrome exists. Sexual activity isn't
usually implicated in spontaneous abortion. Continuing normal activities and
increasing fluid intake fail to address the client's need to see the physician
immediately.

A 35-year-old multigravida at 16 weeks' gestation tells the nurse that she has had
frequent mood swings during this pregnancy. The nurse should suggest that the
patient:
A. seek professional counseling,
B. keep her feelings to herself,
C. try to avoid fatigue and stress.
D. decrease her narcissistic behaviors.

RATIONALE: C. Mood swings are thought to be related to the altered hormonal


levels associated with pregnancy. The nurse should suggest that the patient try to
avoid fatigue and stress because these factors can exacerbate mood swings. The
patient doesn't need professional counseling unless symptoms of psychosis are
present. Telling the patient to keep her feelings to herself or to decrease her
narcissistic behaviors would be inappropriate.

A client in the first trimester complains of nausea every morning and asks about
medicine to prevent it. Which response by the nurse should be most helpful?
A. "Let me tell you about some methods to control nausea without medication."
B. "You shouldn't take medication during pregnancy, especially during the early
weeks."
C. "I'll ask the physician if you can have something."
D. "You'll probably have a lot less nausea in just a few weeks."

RATIONALE: A. Providing ideas to control nausea gives concrete help to the client
without involving drug therapy. Telling her not to take any medications or confirming
that the nausea will probably pass soon may be true in general, but these responses
don't give any help to the woman. These responses may be used as adjunct
explanations along with the nurse's specific suggestions. Drug therapy is inappropriate
without evidence that the nausea is detrimental to the client's health.

53. A nurse is using Doppler ultrasound to assess a pregnant woman. When should the
nurse expect to hear fetal heart tones?
A. 7 weeks
B. 11 weeks
C. 17 weeks
D. 21 weeks

RATIONALE: B. Using Doppler ultrasound, fetal heart tones may be heard between
8 and 12 weeks' gestation. Using a fetoscope, fetal heart tones may be heard between
17 and 20 weeks of gestation.

A nurse is providing care for a pregnant client. The client asks the nurse how she can
best deal with her fatigue. The nurse should instruct her to:
A. take sleeping pills for a restful night's sleep.
B. try to get more rest by going to bed earlier.
C. take her prenatal vitamins.
D. not worry because the fatigue will go away soon.

RATIONALE: B. The client should listen to the body's way of telling her that she
needs more rest and try going to bed earlier. Sleeping pills shouldn't be consumed
prenatally because they can harm the fetus. Vitamins won't take away fatigue. False
reassurance is inappropriate and doesn't help her deal with fatigue now.

61. At what gestational age would a primigravida expect to feel "quickening"?


A. 12 weeks
B. 14 to 16 weeks
C. 18 to 22 weeks
D. By the end of the 26th week

RATIONALE: C. It's important for the nurse to distinguish between a client who's
having her first baby (primigravida) and one who has already had a baby. For the
client who's pregnant for the first time, quickening occurs at around 18 to 22 weeks.
Women who have had children will feel quickening earlier, usually around 14 to 16
weeks, because they recognize the sensations.

62. In assessing a client for pregnancy, the nurse should look for which positive sign?
A. Quickening
B. Amenorrhea
C. Fetal movement felt by the examiner
D. Enlarged uterus

RATIONALE: C. Fetal movement detected by an examiner is an objective, positive


sign of pregnancy. Quickening, a subjective sign of pregnancy experienced by the
woman, isn't as reliable as an independent assessment. Amenorrhea and enlargement
of the uterus can occur in a molar pregnancy or for various reasons unrelated to
pregnancy.

. A pregnant client presents to the emergency department with complaints of back


pain for the second time in a month. She is accompanied by her spouse, who refuses
to let the client speak for herself. When inspecting the painful area, the nurse notes
bruising on the client's lower back. The client's spouse states that the client is clumsy
and falls down the front steps of the house a lot. The nurse should first:
A. ask the physician to order a social service consult while the client is in the
emergency department.
B. inform the client that there are shelters where she can stay if she's being
abused.
C. report the suspected abuse to the local police or authorities.
D. do nothing; this is a private matter between the client and her spouse.

RATIONALE: C. The law requires that nurses report abuse or suspected abuse to
local authorities. The physician may contact social services in addition to referring the
client to community agencies for assistance, but the nurse must still report the abuse.
Doing nothing isn't appropriate. Although the nurse may talk with the client when the
spouse is out of the room and inform her that there are shelters where she can stay,
this action isn't the priority.

65. A pregnant client who's diabetic is at risk for having a large-for-gestational-age


infant because of which of the following?
A. Excess glucose causing reduced placental functioning
B. Insulin acting as a growth hormone on the fetus
C. Maternal dietary intake that's high in calories
D. Excess insulin reducing placental functioning

RATIONALE: B. Insulin acts as a growth hormone on the fetus. Therefore, pregnant


diabetic clients must maintain good glucose control. Large babies are prone to
complications and may have to be delivered by cesarean section. Neither excess
glucose nor excess insulin reduces placental functioning. A high-calorie diet helps
control the mother's disease and doesn't contribute to neonatal size.

A new nurse tells a charge nurse that she isn't comfortable assessing an fetus from the
fetal monitor strip. How should the charge nurse respond?
A. "Don't worry about it. The last shift stated that the strips were normal."
B. "The physician will be back soon. He'll make an assessment."
C. "I'll go with you to interpret this strip, and then we'll look at several others."
D. "I'll reassign you to a client without a monitor."

RATIONALE: C. The charge nurse can improve the new nurse's performance by
taking an active role in increasing her knowledge. A liability occurs when a nurse
takes the word of the previous shift without reassessment. The physician is relying on
the nurse's assessment to notify him if an adverse situation occurs; some time may
elapse before he returns to the unit. Reassignment doesn't help the nurse develop the
skills she needs to work on the unit.

A multigravida at 37 weeks' gestation tells the nurse that she has frequent heartburn.
After providing the client with suggestions for obtaining relief from the heartburn, the
nurse determines that the client has understood the instructions when she says:
A. "I can take a teaspoon of baking soda in water occasionally."
B. "I should eat only three large meals and drink plenty of fluids."
C. "It's all right for me to have a fried hamburger and fries."
D. "I should eat smaller, more frequent meals with fluids."

RATIONALE: D. The client who complains of heartburn should eat smaller, more
frequent meals with fluids. Baking soda in water should be avoided because of the
sodium in baking soda. Large meals and fried foods should also be avoided.
A primipara client is 6 weeks pregnant. At her first prenatal visit, she says to the
nurse, "I just can't believe I'm really pregnant. I hope this baby is a good idea." What
should the nurse make of the client's statement?
A. The client is afraid of pregnancy and birth.
B. The client should have waited until she was committed to having a baby.
C. The client is experiencing normal ambivalence about being pregnant.
D. The client may have problems attaching to the baby after birth.

RATIONALE: C. Ambivalence is normal in the first trimester of pregnancy, even


when a pregnancy is planned and desired. Fear and attachment problems aren't
substantiated by the evidence supplied. Assuming that the client should have waited
to have her baby is judgmental.

A nurse is caring for a client in her 34th week of pregnancy who wears an external
monitor. Which statement by the client should indicate an understanding of the nurse's
teaching?
A. "I'll need to lie perfectly still."
B. "You won't need to come in and check on me while I'm wearing this monitor."
C. "I can lie in any comfortable position, but I should stay off my back."
D. "I know that the external monitor increases my risk of a uterine infection."

RATIONALE: C. The client demonstrates understanding of the external monitor use


when she states she can lie in a comfortable position but avoid being on her back. A
woman should be encouraged to change her position as often as necessary. She
doesn't need to stay perfectly still. However, the monitor may need to be repositioned
after a position change. The nurse still needs to frequently assess and provide
emotional support to a woman in labor who is wearing an external monitor. Because
an external monitor isn't invasive and is worn around the abdomen, it doesn't increase
the risk of uterine infection.

A nurse is teaching a newly pregnant client about nausea associated with prenatal
vitamins. Which point about preventing nausea should the nurse include in her
teaching?
A. Switch brands.
B. Take the vitamin on a full stomach.
C. Take the vitamin with orange juice for better absorption.
D. Take the vitamin first thing in the morning.

RATIONALE: B. Prenatal vitamins commonly cause nausea and taking them on a


full stomach may curb this problem. Switching brands may not be helpful and may be
more costly. Orange juice tends to make pregnant women nauseated. The vitamins
may be taken at night, rather than in the morning, to reduce nausea.

A woman who's 10 weeks pregnant tells the nurse that she's worried about her fatigue
and frequent urination. The nurse should:
A. tell the client that these are normal early pregnancy signs and symptoms.
B. question her further about these signs and symptoms.
C. tell her that she'll need blood work and urinalysis.
D. tell her that she may be excessively worried.
RATIONALE: A. Fatigue and frequent urination are normal early signs and
symptoms of pregnancy that may continue through the first trimester. Questioning the
client about the signs and symptoms is helpful to complete the assessment but won't
reassure her. Prenatal blood work and urinalysis are routine for this situation but don't
address the client's concerns. Telling her that she may be excessively worried isn't
therapeutic

A client in her seventh month of pregnancy has been complaining of back pain and
wants to know what can be done to relieve it. Which response by the nurse is most
effective?
A. "You need to lie down more during the day to get off your feet."
B. "Avoid lifting heavy loads, and try using the pelvic tilt exercise."
C. "Have others pick things up for you so you don't have to bend over so much."
D. "Your back pain will go away after the baby is born."

RATIONALE: B. The pelvic tilt exercise, which can be done standing as well as lying
down, and avoiding lifting heavy loads can greatly relieve back discomfort- As the
pregnancy progresses into the last trimester, women typically develop a "swayback"
curvature of the spine to counterbalance the enlarging fetus. Tilting of the pelvis
aligns the spine, decreasing pressure and back discomfort. Lying down more during
the day may not be possible or convenient for some clients. Also, the supine position
may not be comfortable for some clients and may cause vena cava syndrome
(dizziness on rising and decreased circulation to the fetus). Avoiding bending over as
much may not be realistic for the client's circumstances, nor does it address back pain
as effectively as the pelvic tilt. Telling the client that the pain will go away after the
baby is born doesn't help to relieve the client's discomfort.

A primipara client in her 10th week of pregnancy calls the nurse to say that she's
experiencing slight vaginal bleeding. What's the nurse's best response?
A. "Lie down on your left side and call again if the bleeding worsens."
B. "Save any perineal pads, clots, and tissue and come to the clinic right away."
C. "Avoid sexual intercourse for the next 2 weeks."
D. "Continue your normal activities and increase your fluid intake."

RATIONALE: B. Vaginal bleeding, a sign of threatened abortion, warrants


immediate attention. Saving perineal pads and any matter passed vaginally will make
evaluation more reliable. The left lateral position is important during the last
trimester, when the possibility of vena cava syndrome exists. Sexual activity isn't
usually implicated in spontaneous abortion. Continuing normal activities and
increasing fluid intake fail to address the client's need to see the physician
immediately.

A nurse is caring for a pregnant adolescent. The client is taking an iron supplement.
What should this client drink to increase the absorption of iron?
A. A glass of milk
B. A cup of hot tea
C. A liquid antacid
D. A glass of orange juice
RATIONALE: D. Increasing vitamin C enhances the absorption of iron supplements.
Taking an iron supplement with milk, tea, or an antacid reduces the absorption of
iron.

A community health center physician has confirmed that a 16-year-old adolescent is


at 16 weeks' gestation. The adolescent confides to the nurse that she's afraid to tell her
parents she's pregnant. Which statement best explains the client's feelings?
A. The adolescent lacks a stable relationship with her family.
B. The adolescent fears rejection by her family because of her unplanned
pregnancy.
C. The adolescent's parents may force an abortion.
D. The adolescent can't rely on her family for emotional support.

RATIONALE: B. The adolescent's perception of parental response causes fear of


rejection. Parents' initial reactions to the news of their daughter's pregnancy usually
include shock, anger, shame, guilt, and sorrow. However, in general, adaptation
occurs as the pregnancy progresses, and the adolescent's mother usually becomes her
key support system. No data in this case study indicate abnormally unstable or
nonsupportive family relationships. The evidence given is insufficient to conclude
that the client can't rely on her family for support.

A community health center physician has confirmed that a 16-year-old adolescent is


at 16 weeks' gestation. The adolescent confides to the nurse that she's afraid to tell her
parents she's pregnant. Which statement best explains the client's feelings?
A. The adolescent lacks a stable relationship with her family.
B. The adolescent fears rejection by her family because of her unplanned
pregnancy.
C. The adolescent's parents may force an abortion.
D. The adolescent can't rely on her family for emotional support.

RATIONALE: B. The adolescent's perception of parental response causes fear of


rejection. Parents' initial reactions to the news of their daughter's pregnancy usually
include shock, anger, shame, guilt, and sorrow. However, in general, adaptation
occurs as the pregnancy progresses, and the adolescent's mother usually becomes her
key support system. No data in this case study indicate abnormally unstable or
nonsupportive family relationships. The evidence given is insufficient to conclude
that the client can't rely on her family for support.

A pregnant client is experiencing a thin, odorless, vaginal discharge. What should a


nurse tell her to do to prevent vaginal infections?
A. "Try wearing a panty liner and discarding it after every urination."
B. "When you notice the discharge, take a bath and come into the office."
C. "Don't worry, nothing will happen to you."
D. "You must not be washing thoroughly."

RATIONALE: A. Thin, odorless vaginal discharge is typical during pregnancy.


Keeping the area clean and dry by wearing panty liners will prevent infection. Taking
a bath before an office visit to assess the discharge will wash away the bacteria
needed for examination. Telling the client that she shouldn't worry or that she isn't
keeping herself clean isn't valid and doesn't offer reassurance.
Which instruction should a nurse include in an injury-prevention plan for a pregnant
client?
A. "Wear your seat belt across your tummy."
B. "Position the steering wheel toward your abdomen."
C. "It's OK to start learning a new sport during your pregnancy."
D. "Take rest periods during the day."

RATIONALE: D. The client should be instructed to avoid becoming fatigued and to


take rest periods during the day. Fatigue can lead to injuries. The nurse should instruct
the client to wear a seat belt below the tummy, not across it, and to position the
steering wheel toward her chest, not her abdomen, to prevent injury to the fetus.
Learning a new activity or sport while pregnant can lead to injury.

A client who's been treated for infertility is now pregnant. During a routine ultrasound
at 8 weeks' gestation, she learns that five fetuses are visualized. Concerned that five
infants wouldn't survive and that his wife couldn't handle the stress of the pregnancy,
the client's husband asks the nurse about selective reduction. What is the nurse's best
response?
A. "Why would you consider such a procedure?"
B. "Selective reduction has been used to decrease the possibility of complications
during pregnancy and birth. I'll ask the physician to speak with both of you."
C. "That choice is your wife's because only she is carrying the babies."
D. "You should be glad your wife conceived."

RATIONALE: B. Stating the facts about selective reduction and coordinating a


discussion with the physician are part of the nurse's responsibility to ensure that
clients receive all information needed to make informed decisions. It isn't the role of
the nurse to make judgments or to impart her own beliefs. Nurses must respect that
clients need to be autonomous in decision making.

16. A 15-year-old pregnant adolescent who comes alone to the maternity clinic states
that she and her boyfriend have quit their jobs and run away from home. Which goal
is likely to have the greatest long-term effect on the future well-being of this
adolescent?
A. Promotion of self-esteem
B. Promotion of physical well-being
C. Promotion of family adaptation
D. Facilitation of prenatal education

RATIONALE: A. Adolescent pregnancy is commonly related to problems of


indecision, poor self-image, and egocentrism. If the client can develop self-esteem,
the other goals may follow, and such problems as child abuse, drug abuse, and
unemployment may be overcome or avoided. Physical well-being, family adaptation,
and prenatal education are important goals but not as far-reaching as the development
of self-esteem.

The human embryo grows at a rapid rate. At what gestational age does a single-
chambered heart begin to beat and pump its own blood cells through main blood
vessels?
A. Approximately 8 weeks
B. Approximately 60 days
C. Approximately 5 weeks
D. Approximately 24 days

RATIONALE: D. In the embryo's third week, the heart becomes the most advanced
organ. Around the 24th day, a single-chambered heart forms just outside the embryo's
body cavity and begins beating a regular rhythm, pushing its own primitive blood
cells through the main blood vessels.

19. A pregnant client is receiving heparin. What should the nurse assess every shift?
A. Change in fetal activity and position
B. Increase in blood pressure and temperature
C. Any signs of preterm labor and bleeding from any orifice
D. Homans' sign or periorbital edema

RATIONALE: C. Potential complications of heparin therapy are preterm labor and


maternal hemorrhage. Assessment of fetal activity is important, but fetal position isn't
significant at this time. Heparin doesn't increase blood pressure or temperature. The
client should be assessed for Homans* sign if she's at risk for deep vein thrombosis;
periorbital edema is assessed in gestational hypertension.

You might also like